Rau's IAS Prelims 2024 Test 44 With Solution

You might also like

Download as pdf or txt
Download as pdf or txt
You are on page 1of 74

https://upscpdf.com/ https://upscpdf.

com/

https://upscpdf.com/ https://upscpdf.com/
https://upscpdf.com/ https://upscpdf.com/

INSTRUCTIONS

1. IMMEDIATELY AFTER THE COMMENCEMENT OF THE EXAMINATION, YOU SHOULD CHECK


THAT THIS TEST BOOKLET DOES NOT HAVE ANY UNPRINTED OR TORN or MISSING PAGES
OR ITEMS, ETC. IF SO, GET IT REPLACED BY A COMPLETE TEST BOOKLET.

2. Please note that it is the candidate’s responsibility to encode and fill in the Student ID and
Test ID carefully and without any omission or discrepancy at the appropriate places in the
OMR Answer Sheet. Any omission/discrepancy will render the Answer Sheet liable for rejection.

3.

4. This Test Booklet contains 100 items (questions). Each item is printed in English Only. Each
item comprises four responses (answers). You will select the response which you want to mark
on the Answer Sheet. In case you feel that there is more than one correct response, mark the
response which you consider the best. In any case, choose ONLY ONE response for each item.

5. You have to mark all your responses ONLY on the separate Answer Sheet (OMR sheet) provided.
Read the directions in the Answer Sheet.

6. All items carry equal marks.

7. Before you proceed to mark in the Answer Sheet the response to various items in the Test
booklet, you have to fill in some particulars in the Answer Sheet as per instructions contained
therein.

8. After you have completed filling in all your responses on the Answer Sheet and the examination
has concluded, you should hand over to the Invigilator only the Answer Sheet. You are
permitted to take away with you the Test Booklet.

9. Penalty for wrong answers:

THERE WILL BE PENALTY FOR WRONG ANSWERS MARKED BY A CANDIDATE IN THE


OBJECTIVE TYPE QUESTION PAPERS.

(i) There are four alternatives for the answer to every question. For each question for which
a wrong answer has been given by the candidate, one-third of the marks assigned to that
question will be deducted as penalty.

(ii) If a candidate gives more than one answer, it will be treated as a wrong answer even if
one of the given answers happens to be correct and there will be same penalty as above
to that question.

(iii) If a question is left blank, i.e., no answer is given by the candidate, there will be no
penalty for that question.

https://upscpdf.com/ https://upscpdf.com/
https://upscpdf.com/ https://upscpdf.com/

1. Which of the following archaeological sites 4. With reference to the Kushana rulers,
of the Kumaon Hills is renowned for its pre- consider the following statements:
historic paintings, characterized by three 1. Many Kushana rulers adopted the title
distinct categories – depictions of the ‘Devaputra’.
humans, animals and geometric patterns in
2. The gold coins issued by the Kushana
white, black and red ochre?
rulers were virtually identical in weight
Additionally, the interesting scenes of with those issued by the contemporary
hand-linked dancing human figures were Parthian rulers of Iran.
depicted here, along with a long-snouted
3. The Rabatak inscription relates to the
animal, a fox and a multiple legged lizard.
Kushana dynasty.
(a) Lakhudiyar
How many of the above statements is/are
(b) Bhimbetka
correct?
(c) Kupgallu
(a) Only one
(d) Bainoli
(b) Only two
(c) All three
2. With reference to the Buddhist sites,
(d) None
consider the following statements:
1. The relics of Budhha have been found
in the Devni Mori Stupa, Madhya 5. Consider the following Gods:
Pradesh. 1. Kubera
2. Independent Buddha images are found 2. Varuna
at Amaravati and Nagarjunkonda. 3. Indra
3. The Sannati Stupa is situated on the 4. Yama
Bhima river. How many of the above Gods is/are
How many of the above statements is/are mentioned in the Prayaga Prashasti?
correct? (a) Only one
(a) Only one (b) Only two
(b) Only two
(c) Only three
(c) All three
(d) All four
(d) None

6. The Greek word “Erythraean” indicates


3. Consider the following statements: which of the following seas?
1. Phamsana buildings in the Nagara (a) The Black Sea
order tend to be broader and shorter
(b) The Caspian Sea
than the Latina ones.
(c) The Mediterranean Sea
2. The Dashavatara temple is in the
Panchayatana style of architecture. (d) The Red Sea
3. The Lakshmana temple of Khajuraho is
dedicated to Shiva. 7. Consider the following travellers:
4. The Pandrethan temple in Kashmir is 1. William Finch
influenced by the post-Gupta aesthetics 2. Joseph Tieffenthaler
of heavy carving. 3. Joannes de Laet
How many of the above statements is/are How many of the above travellers visited
correct? “Ayodhya”?
(a) Only one
(a) Only one
(b) Only two
(b) Only two
(c) Only three
(c) All three
(d) All four
(d) None

RAUSIAS-0673 1

https://upscpdf.com/ https://upscpdf.com/
https://upscpdf.com/ https://upscpdf.com/

8. With reference to Krishnadeva Raya, 2. During the Mughal period, the term
consider the following statements: Muzarian was used to denote a peasant.
1. Krishnadeva Raya composed Which of the statements given above is/are
Amuktamalyada on statecraft in correct?
Sanskrit. (a) 1 only
2. He belonged to the Tuluva dynasty. (b) 2 only
3. The ‘Raichur doab’ was acquired during (c) Both 1 and 2
his period.
(d) Neither 1 nor 2
How many of the above statements is/are
correct?
12. Which one of the following statements,
(a) Only one
regarding the typical ‘Subsidiary Alliance
(b) Only two
Treaty’ during the colonial rule in India, is
(c) All three
incorrect?
(d) None
(a) The East India Company had the right
to deal with the external affairs of the
9. With reference to the ‘market control’ and state, and the state could make no wars
the ‘agrarian policy’ of Alauddin Khalji, after signing the treaty.
which of the following statements is
(b) A bigger state was to cede territory in
incorrect?
full sovereignty for the maintenance of
(a) Good quality horses could be sold only
the army commanded by the British
to the state.
officer.
(b) The merchants were commandeered to
(c) The Company, after the signing of the
transport grains from villages to Delhi.
treaty, could interfere in the internal
(c) The land revenue was raised to half of
matters of the state.
the produce.
(d) The state was required to consult the
(d) An official known as 'Barid' was
Company before employing any
appointed as the superintendent of the
European in its service.
market.

10. With reference to Sher Shah’s 13. In the context of Indian history, consider
administration, which of the following the following statements:
statements is incorrect? 1. The Prarthana Samaj and the Aligarh
(a) The Minister in charge of the Diwan-i- Movements were revivalist in nature.
Insha was to keep himself in touch with 2. The Dharma Sabha was formed by
the ambassadors and the envoys. Radhakant Deb to counter the
(b) The names on the coins were given in teachings of the Brahmo Samaj.
Devanagari script. 3. Dayanand Saraswati strongly
(c) The Sarais were used as Dak Chaukis. condemned the ‘Vedic notion of the
(d) The Pargana was the main unit for Varna system’.
revenue collection. How many of the above statements is/are
correct?
11. With reference to the Mughal empire, (a) Only one
consider the following statements: (b) Only two
1. Pahi-Kashtas were the resident (c) All three
peasants of the village in which they
(d) None
held their lands.

RAUSIAS-0673 2

https://upscpdf.com/ https://upscpdf.com/
https://upscmaterial.online/

https://upscpdf.com/ https://upscpdf.com/
14. “A Night of Eternal Gloom for India” was 17. With reference to the Swatantra Party, the
written by Nabin Chandra Sen in which of first authentic all-India secular
the following contexts? conservative party, consider the following
(a) The Battle of Plassey statements:
(b) The First Battle of Panipat 1. C. Rajagopalachari and Minoo Masani
(c) The Annexation of Awadh were the members of the Party.
(d) The Battle of Wandiwash 2. The Party supported the active role of
the State in economic development.
Which of the statements given above is/are
15. With reference to the Nehru Report,
correct?
consider the following statements:
(a) 1 only
1. The Report visualized joint electorates
with time-bound reservation of seats for (b) 2 only
the minorities. (c) Both 1 and 2
2. It recommended the ‘right to free (d) Neither 1 nor 2
elementary education' as a
Fundamental Right. 18. With reference to the ‘Captive Elephant
3. It recommended that the distribution of (Transfer or Transport) Rules, 2024’,
the provinces should take place on a consider the following statements:
linguistic basis. 1. It authorizes the Chief Wildlife Warden
How many of the above statements is/are of the states and the UTs to permit or
correct? reject the transfer of the captive
(a) Only one elephants.
(b) Only two 2. The transfer of the captive elephants
(c) All three will not be permissible unless the
genetic profile of the animal has been
(d) None
entered in the electronic monitoring
application of the Ministry of
16. Consider the following events with Environment, Forest and Climate
reference to the Non-Cooperation Change.
Movement (1920-22):
3. The Rules granted exemptions for the
1. Formation of the Indian National transfer of the captive elephants for
Liberation Federation. ‘religious and other purposes’.
2. Foundation of many educational How many of the above statements is/are
institutions, like the Banaras Hindu correct?
University.
(a) Only one
3. Creation of the Tilak Swaraj Fund.
(b) Only two
4. Formation of the Satyagraha Sabha by
(c) All three
Gandhi.
(d) None
How many of the above events did not take
place during the Non-Cooperation
Movement? 19. With reference to the ‘snow leopard’,
consider the following statements:
(a) Only one
1. The highest number of snow leopards in
(b) Only two
India is found in Uttarakhand.
(c) Only three
2. It is listed as ‘Endangered’ in the
(d) All Four
International Union for Conservation of
Nature Red List.

RAUSIAS-0673 3
https://upscpdf.com/ https://upscpdf.com/
https://t.me/visioniastestseries2024
https://upscmaterial.online/

https://upscpdf.com/ https://upscpdf.com/
3. Adult snow leopards are solitary. 2. West Bengal and Bihar are participating
4. Snow leopards cannot roar. in the TOFI.
How many of the above statements is/are 3. The TOFI intends to create inclusive
correct? economic opportunities and enhance
(a) Only one ecosystem services, including carbon
(b) Only two sequestration.
(c) Only three How many of the above statements is/are
(d) All four correct?
(a) Only one
20. Consider the following species:
(b) Only two
1. Box turtles
(c) All three
2. Painted turtles
3. Alligators (d) None
4. Bats
How many of the above species goes/go 23. Consider the following species:
through ‘brumation’? 1. Allies
(a) Only one
2. Crustaceans
(b) Only two
3. Mosses
(c) Only three
4. Sea slug
(d) All four
How many of the above species

21. With reference to the ‘Global Framework on comes/come under ‘plants’?


Chemicals’, consider the following (a) Only one
statements: (b) Only two
1. The aim of the Framework is to phase- (c) Only three
out lethal agricultural pesticides by
(d) All four
2035.
2. The Global Framework on Chemicals
Fund will be financed from 24. Consider the following statements:
contributions from all stakeholders, 1. Species diversity is generally the highest
excluding the private sector.
in the tropics and decreases towards
Which of the statements given above is/are the poles.
correct?
2. Rauwolfia vomitoria, a medicinal plant,
(a) 1 only
is found in different Himalayan ranges.
(b) 2 only
3. The Eastern Ghats have a greater
(c) Both 1 and 2
amphibian species diversity than the
(d) Neither 1 nor 2
Western Ghats.

22. With reference to the ‘Trees Outside Forests How many of the above statements is/are
in India’ (TOFI), consider the following correct?
statements: (a) Only one
1. The TOFI is a joint programme of the (b) Only two
United Nations Environment
(c) All three
Programme and the Ministry of
Environment, Forest and Climate (d) None
Change, the Government of India.

RAUSIAS-0673 4
https://upscpdf.com/ https://upscpdf.com/
https://t.me/visioniastestseries2024
https://upscmaterial.online/

https://upscpdf.com/ https://upscpdf.com/
25. Consider the following pairs: 28. With reference to decomposition, consider
S.N. Wetlands State/UT the following statements:
1. The process in which bacterial and
1. The Karaivetti Bird Lakshadweep
fungal enzymes degrade detritus into
Sanctuary
simpler inorganic substances is known
2. The Ankasamudra Madhya as catabolism.
Bird Conservation Pradesh 2. In a particular climatic condition, the
Reserve decomposition rate is quicker, if
3. The Aghanashini Karnataka detritus is rich in lignin and chitin.
Estuary Which of the statements given above is/are
How many of the above pairs is/are correct?
correctly matched? (a) 1 only
(a) Only one (b) 2 only
(b) Only two (c) Both 1 and 2
(c) All three (d) Neither 1 nor 2
(d) None
29. Consider the following statements:
26. With reference to the Carbon Border 1. Among the states in India, the highest
Adjustment Mechanism (CBAM), consider elephant population is in Karnataka.
the following statements: 2. At present, West Bengal has the
1. The CBAM is a measure of the maximum number of elephant
European Union and USA that corridors across the country.
encourages the global industry to 3. The Sigur Elephant Corridor connects
embrace greener and more sustainable the Western Ghats to the Eastern
technologies. Ghats.
2. In its transitional phase, the CBAM will How many of the above statements is/are
apply only to the imports of cement, correct?
iron and steel, aluminium, fertilizers, (a) Only one
electricity and hydrogen.
(b) Only two
3. India has challenged the CBAM before
(c) All three
the World Trade Organization under the
(d) None
‘Special and Differential Treatment
Provisions’.
How many of the above statements is/are 30. Consider the following statements
correct? regarding Seagrasses:
(a) Only one 1. Seagrasses are found in salty and
brackish waters at a shallow depth.
(b) Only two
2. Seagrasses are called the foundational
(c) All three
plant species or the ecosystem
(d) None
engineers.
3. The largest seagrass bed in the country
27. The ‘Montecincla’ and the ‘Sholicola’, some is in the Gulf of Khambhat.
of the oldest bird species, are found in
How many of the above statements is/are
which of the following regions?
correct?
(a) The Eastern Ghats
(a) Only one
(b) The Western Himalayas
(b) Only two
(c) The Eastern Himalayas
(c) All three
(d) The Western Ghats
(d) None

RAUSIAS-0673 5
https://upscpdf.com/ https://upscpdf.com/
https://t.me/visioniastestseries2024
https://upscmaterial.online/

https://upscpdf.com/ https://upscpdf.com/
31. With reference to the Scheduled Tribes and 3. The Milam The Kumaon
Other Traditional Forest Dwellers Glacier Himalayas
(Recognition of Forest Rights) Act, 2006, 4. The Pensilungpa The Garhwal
consider the following statements: Glacier Himalayas
1. The community forest resource rights
How many of the above pairs is/are
include ‘Nistar’ and the rights over non- correctly matched?
timber forest products.
(a) Only one
2. The Gram Sabha has the authority to (b) Only two
adopt the local traditional practices of
(c) Only three
forest conservation and management
(d) All four
within the community forest resource
boundary.
34. Consider the following statements with
Which of the statements given above is/are
regard to the de-notification of the National
correct? Parks and the Tiger Reserves in India:
(a) 1 only 1. The de-notification of a National Park in
(b) 2 only India requires the approval of the
(c) Both 1 and 2 Supreme Court.
(d) Neither 1 nor 2 2. The de-notification of a Tiger Reserve is
done by the National Tiger Conservation
Authority.
32. With reference to mangroves, consider the
3. The Pobitora Wildlife Sanctuary has
following statements:
been de-notified as a Wildlife
1. Mangroves are halophytes found in the Sanctuary.
inter-tidal zones in the tropical and How many of the above statements is/are
sub-tropical zones. correct?
2. Pneumatophores allow the mangroves (a) Only one
to absorb gases directly from the (b) Only two
atmosphere. (c) All three
3. Globally, Brazil has the largest (d) None.
mangrove cover.
How many of the above statements is/are 35. Which of the following statements is/are
correct? correct regarding ‘Fumaroles’?
(a) Only one 1. These are the vents in the Earth’s
(b) Only two surface from which steam and volcanic
(c) All three gases are emitted.

(d) None 2. Brightly coloured rocks are formed in


the areas having such features.
Select the correct answer using the code
33. Consider the following pairs:
given below:
S.N. Glacier Location (a) 1 only
1. The Dokriani The Karakoram (b) 2 only
Glacier Range (c) Both 1 and 2
2. The Drang- The Zanskar (d) Neither 1 nor 2
Drung Glacier Range

RAUSIAS-0673 6
https://upscpdf.com/ https://upscpdf.com/
https://t.me/visioniastestseries2024
https://upscmaterial.online/

https://upscpdf.com/ https://upscpdf.com/
36. Zeugens, yardangs and dreikanters are 40. Consider the following statements:
associated with which of the following? 1. Carbon dioxide is opaque to the
(a) Glacial landforms incoming solar radiation and
(b) Aeolian landforms transparent to the outgoing terrestrial
(c) Marine landforms radiation.
(d) Karst landforms
2. Oxygen and water vapour are found
only upto 40 km from the surface of the
37. Consider the following statements:
Earth.
1. Planktons are the diverse collection of
Which of the statements given above is/are
organisms found in water, that are
unable to propel themselves against a correct?
current. (a) 1 only
2. Benthos are the living organisms that (b) 2 only
are able to swim and move
(c) Both 1 and 2
independently of currents.
(d) Neither 1 nor 2
3. Nektons is the community of organisms
that live on, in, or near the seabed.
How many of the above statements is/are 41. Which one of the following is the correct
correct? sequence of the Himalayan peaks, from
(a) Only one west to east direction?
(b) Only two (a) Kamet - Nanda Devi – Kanchenjunga -
(c) All three Annapurna
(d) None
(b) Kamet - Nanda Devi - Annapurna -
Kanchenjunga
38. Which of the following types of lakes is
(c) Nanda Devi – Kamet - Annapurna -
formed by tectonic activity and is typically
characterized by a deep basin with steep Kanchenjunga
sides? (d) Nanda Devi – Kamet – Kanchenjunga -
(a) Oxbow lake Annapurna
(b) Glacial lake
(c) Crater lake 42. With reference to natural vegetation in
(d) Lagoon India, consider the following statements:
1. Tropical evergreen forests are well
39. Consider the following statements stratified.
regarding temperature inversion:
2. Amaltas and kusum are found in the
1. Surface inversion promotes instability
in the lower layers of the atmosphere. tropical evergreen forests.
2. Dense fogs in the mornings in winter 3. The montane temperate forests are
season are common occurrences due to called ‘Sholas’ in the Anaimalai and the
temperature inversion. Palani hills.
3. It allows the wind to move and disperse How many of the above statements is/are
pollutants. correct?
How many of the above statements is/are
(a) Only one
correct?
(a) Only one (b) Only two
(b) Only two (c) All three
(c) All three (d) None
(d) None

RAUSIAS-0673 7
https://upscpdf.com/ https://upscpdf.com/
https://t.me/visioniastestseries2024
https://upscmaterial.online/

https://upscpdf.com/ https://upscpdf.com/
43. Consider the following statements: through the bites of
1. The Sankosh river is a tributary of the the infected
Brahmaputra river. mosquitoes.
2. The Sukapaika river is a distributary of How many of the above pairs is/are
the Mahanadi river. correctly matched?
3. The Indravati river is a tributary of the (a) Only one
Krishna river. (b) Only two
How many of the above statements is/are (c) All three
correct? (d) None
(a) Only one
(b) Only two 46. Consider the following statements with
(c) All three reference to the Gaganyaan mission:
(d) None 1. The Gaganyaan mission envisages the
launching of a crewed spaceflight to the
44. Consider the following statements Geostationary Orbit.
regarding the Deccan Traps in India: 2. Geosynchronous Satellite Launch
1. They were formed due to the Vehicle Mark II will serve as the launch
outpourings of extremely mobile basalt vehicle for the mission.
lava from fissures and cracks. 3. The Spaceflight Programme will be
2. They cover parts of Kuchchh, coordinated by the Human Space Flight
Saurashtra, Maharashtra, the Malwa Centre.
Plateau and northern Karnataka. How many of the above statements is/are
3. The thickness of the Deccan Traps correct?
gradually increases from Kuchchh to (a) Only one
Amarkantak. (b) Only two
How many of the above statements is/are (c) All three
correct? (d) None
(a) Only one
(b) Only two 47. Consider the following statements with
(c) All three reference to the ‘messenger RNA’ (mRNA):
(d) None 1. The structure or composition of the
nucleotide bases present in the mRNA
45. Consider the following pairs: molecule can be chemically altered.
S.N. Disease Description 2. The alteration of bases can help in
stabilizing the mRNA and makes it last
1. Human Infects the
longer.
papillomavirus reproductive tract
3. mRNA alteration can help prevent
of women, and is
unwanted immune reactions in the
not able to infect
body.
men.
How many of the above statements is/are
2. Pompe disease Inherited blood
correct?
disorder that affects
(a) Only one
haemoglobin.
(b) Only two
3. Kala-azar A parasitic disease,
(c) All three
spreads to the
humans mainly (d) None

RAUSIAS-0673 8
https://upscpdf.com/ https://upscpdf.com/
https://t.me/visioniastestseries2024
https://upscmaterial.online/

https://upscpdf.com/ https://upscpdf.com/
48. Consider the following statements with difficult for them to
reference to India’s Prototype Fast Breeder cancel it.
Reactor (FBR): 2. Drip Involves hiding
1. India envisages using the FBR at the Pricing additional costs from
third stage of its nuclear energy the consumers, until
programme. they are already
2. The FBR uses the Uranium-Plutonium committed to making a
Mixed Oxide fuel. purchase.
3. The FBR can aid in the conversion of
3. False Involves guilt as a way
fertile Thorium 232 to fissile Uranium
Urgency to make the
233.
consumers adhere to a
4. The FBR recycles the waste from the
particular belief or
Pressurized Heavy Water Reactor by
view-point.
utilizing it as a fuel.
How many of the above statements is/are 4. Bait and Practice of advertising
correct? Switch a particular outcome,
based on the user’s
(a) Only one
action, but deceptively
(b) Only two
serving an alternate
(c) Only three
outcome.
(d) All four
How many of the above pairs is/are
correctly matched?
49. Consider the following statements:
(a) Only one
1. Buckminsterfullerene is an allotrope of
(b) Only two
carbon which is not found naturally.
2. Fullerene is a one-atom thick sheet of (c) Only three
carbon that can be rolled to form (d) All four
carbon nanotubes.
3. Fullerene has energy storage 51. Consider the following statements with
applications and can be used in organic reference to the ‘stem cell’:
solar cells. 1. The fertilized cell or zygote is a
How many of the above statements is/are pluripotent stem cell.
correct? 2. Pluripotent stem cells are capable of
(a) Only one differentiating into the extraembryonic
(b) Only two cells.
(c) All three 3. Unipotent stem cells have the self-
(d) None regenerative potential.
How many of the above statements is/are
50. Consider the following pairs: correct?

S.N. Dark Definition (a) Only one


patterns (b) Only two

1. Basket Makes it easy for the (c) All three


Sneaking consumers to sign-up (d) None
for a service, but

RAUSIAS-0673 9
https://upscpdf.com/ https://upscpdf.com/
https://t.me/visioniastestseries2024
https://upscmaterial.online/

https://upscpdf.com/ https://upscpdf.com/
52. Which of the following best describes ‘SAR- 3. The Anusandhan National Research
Arcs’, often mentioned in the news? Foundation will undertake an annual
(a) Aurora like phenomena which result in survey of the outcomes of the scientific
the display of vivid shades of red colour. research in India.
(b) A satellite to study the ‘Coronal Mass How many of the above statements is/are
Ejection’. correct?
(c) A ‘dark sky preserve’ that restricts (a) Only one
artificial light pollution. (b) Only two
(d) Aurora observed at the Poles. (c) All three
(d) None
53. Which of the following are the applications
of the ‘quantum dots’?
56. Consider the following statements:
1. Bio-sensors
1. The State cannot regulate or control
2. Flexible electronics
admissions in the minority educational
3. Targeted cancer treatment
institutions, so as to compel them to
4. High-quality visual display give up a share of the available seats to
Select the correct answer using the code the reserved categories.
given below:
2. The ‘Private Unaided Educational
(a) 1 and 3 only Institutions’ are allowed to make profits,
(b) 2 and 3 only but not profiteering.
(c) 1, 2 and 3 only Which of the statements given above is/are
(d) 1,2,3 and 4 correct?
(a) 1 only
54. Which of the following statements best (b) 2 only
describes the Picoflare jets?
(c) Both 1 and 2
(a) Dense remnants of supermassive stars
(d) Neither 1 nor 2
that have exploded as supernovae.
(b) Slow and gradual emissions of radio
waves from pulsars. 57. Which of the following is the best
(c) Small jets of charged particles expelled description of the term ‘Constitutional
intermittently from the Sun’s corona. Tort’, which was recently in the news?
(d) Intense bursts of gamma rays from the (a) It is a violation of one’s constitutional
distant galaxies. rights by a government employee.
(b) It is a phenomenon of overturning an
55. Consider the following statements about amendment by the Apex Court.
the Anusandhan National Research (c) It is the breach of an agreement by the
Foundation Act, 2023: Federal Government.
1. The Governing Board of the (d) It is a criminal offence against the
Anusandhan National Research constitutional office by an individual.
Foundation will be headed by the Union
Minister of Science and Technology.
58. Consider the following statements with
2. The Executive Council of the reference to the Union Territories:
Anusanshan National Research
1. The Constitution (Eighteenth
Foundation will be headed by the
Amendment) Act, 1966, empowered the
Secretary of the Union Department of
Parliament to increase or diminish the
Science and Technology.
area, alter the boundaries or the name
of the Union Territories.

RAUSIAS-0673 10
https://upscpdf.com/ https://upscpdf.com/
https://t.me/visioniastestseries2024
https://upscmaterial.online/

https://upscpdf.com/ https://upscpdf.com/
2. A new territory, as a Union Territory, How many of the above statements is/are
can only be admitted and established correct?
through an amendment of the (a) Only one
Constitution under Article 368. (b) Only two
3. An amendment of the Constitution
(c) All three
under Article 368 is necessary to
(d) None
convert a State into one or more Union
Territory/Territories.
How many of the above statements is/are 61. With reference to the Indian Constitution,
correct? consider the following pairs:
(a) Only one S.N. The Minimum support
(b) Only two Objective of required in both
(c) All three the the Houses of
(d) None Resolution Parliament
1. Impeachment Majority of not less
59. Consider the following statements: of the than two-thirds of
1. Prorogation terminates a session and President of the then members,
does not constitute an interruption in India present and voting.
the continuity of life of the Lok Sabha.
2. Removal of a Majority of not less
2. Once the Lok Sabha has been
Judge of the than two-thirds of
adjourned sine die, it cannot be recalled
Supreme the members of
for sitting by the Speaker.
Court that House present
3. Usually, Prorogation follows the
and voting only.
adjournment of the sitting of the
House sine die. 3. Creation of a Majority of not less
How many of the above statements is/are new All-India than two-thirds of
correct? Service the members,
(a) Only one present and voting.
(b) Only two How many of the above pairs is/are
(c) All three correctly matched?
(d) None (a) Only one
(b) Only two
60. Consider the following statements with (c) All three
respect to the Election Symbols
(d) None
(Reservation and Allotment) Order, 1968:
1. It provides for the classification of the
political parties as State, National or 62. With reference to the investigating agencies
unrecognised political party. in India, consider the following pairs:
2. A recognised political party shall either S.N. Agency Parent
be a National party or a State party. Ministry
3. The Election Commission is empowered 1. The National The Ministry of
to adjudicate the disputes in relation to Investigation Home Affairs
the splinter groups or rival sections of a Agency
recognised political party, and the
decision of the Commission shall be 2. The Serious The Ministry of
binding on all such rival sections or Fraud Finance
groups. Investigation
Office

RAUSIAS-0673 11
https://upscpdf.com/ https://upscpdf.com/
https://t.me/visioniastestseries2024
https://upscmaterial.online/

https://upscpdf.com/ https://upscpdf.com/
3. The Central The Ministry of Which of the statements given above is/are
Bureau of Personnel, Pub correct?
Investigation lic (a) 1 only
Grievances and (b) 2 only
Pensions (c) Both 1 and 2
How many of the above pairs is/are (d) Neither 1 nor 2
correctly matched?
(a) Only one 65. With reference to the 73rd Constitutional
(b) Only two Amendment Act, 1992, consider the
following statements:
(c) All three
1. A Panchayat, constituted upon the
(d) None
dissolution of a Panchayat before the
expiration of its duration, shall
63. In the pretext of torture, policy custody and continue for the full term of five years.
the rights of the accused, consider the 2. The 73rd Constitution Amendment Act
following statements: limits the amount of professional tax to
1. A person, who is arrested, shall be be imposed on a person to Rs. 2,500.
informed of the grounds of arrest and 3. It provides for the reservation of the
shall have a right to consult and to be seats for the ‘backward classes’ in every
defended by a legal practitioner of his Panchayat, in proportion to their
choice. population.
2. No person shall be prosecuted and How many of the above statements is/are
punished for the same offence more correct?
than once, and no person accused of (a) Only one
any offence shall be compelled to be a (b) Only two
witness against himself. (c) All three
3. The confession made by an arrested (d) None
person in police custody, because of
extreme torture, can be used as 66. Consider the following statements:
evidence placed before the magistrate to
1. Under the Mont–Ford Reforms of 1919,
prove the guilt. the first Auditor -General was
How many of the above statements is/are appointed.
correct? 2. The Constitution of India provides for
(a) Only one the appointment of the Comptroller and
(b) Only two Auditor-General of India at the Central-
(c) All three level and other Comptroller and
Auditor-Generals at the State-level.
(d) None
3. The Comptroller and Auditor-General
shall not be eligible for further office,
64. Consider the following statements: either under the Government of India,
1. The office of the Deputy Chief Minister or under the Government of any State,
is mentioned in the Constitution of after he has ceased to hold his office.
India. How many of the above statements is/are
2. In the ‘Official Table of Precedence’, the correct?
Deputy Chief Minister is ranked the (a) Only one
same as the Cabinet Minister of the (b) Only two
State. (c) All three
(d) None

RAUSIAS-0673 12
https://upscpdf.com/ https://upscpdf.com/
https://t.me/visioniastestseries2024
https://upscmaterial.online/

https://upscpdf.com/ https://upscpdf.com/
67. With reference to the Governor of a State in 3. A Bill which contains provision dealing
India, consider the following statements: with the receipt of money on account of
1. In the summoning of the Legislative the Public Account of India is a Money
Assembly by the Governor, three Bill.
months shall not intervene between its 4. A Financial Bill can be introduced
last sitting in one session and the date without the recommendation of the
appointed for its first sitting in the next President of India.
session.
How many of the above statements is/are
2. The Legislative Assembly is obligated to correct?
provide in its rules for the procedure for
(a) Only one
the allotment of time for discussing the
Governor’s address. (b) Only two
3. The Constitution makes provision to (c) Only three
meet the contingency arising out of the (d) All four
Governor’s unavoidable absence due to
illness on the occasion of the opening of 70. Consider the following statements about
the Legislature. the Parliamentary Privileges under the
How many of the above statements is/are Constitution of India:
correct?
1. The Privileges are the powers which are
(a) Only one necessary for the conduct of business of
(b) Only two each House of the Parliament.
(c) All three 2. The freedom of speech in the Parliament
(d) None is absolute.
3. Whether a claim to constitutional
68. With reference to the Constitution of India, privilege in a particular case conforms
consider the following statements: to the parameters of the Constitution is
1. The State Legislature cannot legislate amenable to judicial review.
on the State List while a Proclamation How many of the above statements is/are
of Emergency is in operation. correct?
2. The Parliament has the power to make (a) Only one
any law in the State List for
(b) Only two
implementing any international treaties.
(c) All three
Which of the statements given above is/are
correct? (d) None
(a) 1 only
(b) 2 only 71. Which among the following best describes
(c) Both 1 and 2 the concept of Synthetic CBDC?
(d) Neither 1 nor 2 (a) A form of CBDC issued by the Central
Bank, which is based on the
Distributed Ledger Technology.
69. With reference to the Financial Bill and the
Money Bill, consider the following (b) A form of CBDC issued by the Central
statements: Bank for cross-border payments.
1. Both the Financial Bill and the Money (c) A form of CBDC which has been
Bill originate only in the Lok Sabha. programmed to be used for certain
2. A deadlock between the Lok Sabha and selected transactions only.
the Rajya Sabha over a Financial Bill (d) A form of CBDC which is issued by the
will have to be resolved through the banks, but backed by the Central Bank
procedure of a ‘joint session’. Reserves.

RAUSIAS-0673 13
https://upscpdf.com/ https://upscpdf.com/
https://t.me/visioniastestseries2024
https://upscmaterial.online/

https://upscpdf.com/ https://upscpdf.com/
72. Consider the following scenario: 75. Which of the following statements, about
The RBI increases or decreases the ‘Policy borrowings made by the State
Rates’ to control inflation or to promote Governments, is/are correct as per the
growth. This forces the banks to change the Constitution?
rate of interest on the deposits, accordingly. 1. The Centre can impose restrictions on
If you have Rs. 10,000 in the form of cash, the States’ borrowing limits, if there is
any outstanding loan pending with the
then, your opportunity cost of holding cash
Centre.
(a) increases with decrease in the interest
2. The Fiscal Responsibility and Budget
rates.
Management (FRBM) Act, 2003 enacted
(b) decreases with increase in the interest
by Parliament mandates a ceiling on the
rates. guarantees given by the centre and
(c) increases with increase in the interest state governments.
rates. Select the correct answer using the code
(d) remains the same despite change in the given below:
interest rates. (a) 1 only
(b) 2 only
73. Recently, the ‘Banking System Liquidity’ (c) Both 1 and 2
has reached its lowest level in the last 8 (d) Neither 1 nor 2
years. How many, among the following,
could lead to decrease in the ‘Banking 76. How many of the following statements
System Liquidity’ in the Indian economy? is/are correct about the Competition
1. An increase in demand for currency (Amendment) Act, 2023?
2. Forex intervention by the Reserve Bank 1. The Act prohibits any mergers and
of India (RBI) acquisitions whose transactions value is
3. Decrease in the ‘Credit-Deposit Ratio’ above Rs. 2,000 crores.
Select the correct answer using the code 2. The Act is applicable only to the
given below: enterprises engaged in identical or
similar businesses.
(a) Only one
3. The Act enabled the CCI to reduce
(b) Only two
penalty for an enterprise in exchange of
(c) All three
information on other anti-competitive
(d) None agreements in the market.
Select the correct answer using the code
74. Which among the following best describes given below:
the situation of “Connected Lending” in the (a) Only one
banking sector? (b) Only two
(a) A bank sanctioning loans to another (c) All three
bank. (d) None
(b) A bank partnering with an NBFC to
provide loans to the industries. 77. Which of the following statements is/are
(c) A bank sanctioning loans to the Board correct about the ‘future contract’ in a
of Directors, or their family members. ‘commodity derivative market’?
(d) A bank partnering with another bank to 1. A ‘Future’ is a derivative instrument
give loans to its customers. whose value is based on the value of the
underlying commodity.

RAUSIAS-0673 14
https://upscpdf.com/ https://upscpdf.com/
https://t.me/visioniastestseries2024
https://upscmaterial.online/

https://upscpdf.com/ https://upscpdf.com/
2. In the future contract, the buyer has no 80. Which of the following statements about
obligation to buy the assets. Buy back of shares as a strategy of
Select the correct answer using the code disinvestment is/are correct?
given below: 1. Buy back of share reduces the share of
(a) 1 only government in the Public Sector Units.
(b) 2 only 2. It does not alter the total equity capital
(c) Both 1 and 2 of the company.
(d) Neither 1 nor 2 Select the correct answer using the code
given below:
78. Consider the following statements: (a) 1 only
1. While the banks maintain the FCNR
(b) 2 only
accounts in foreign currency, the NRE
(c) Both 1 and 2
accounts are essentially rupee-
denominated accounts. (d) Neither 1 nor 2
2. While the deposits made in the FCNR
accounts are partially repatriable, the 81. Consider the following statements about
NRE deposits are fully repatriable. the WTO's General Agreement on Trade in
3. The interest earned on both the FCNR Services (GATS):
account and the NRE account is tax-
1. India is a signatory of the GATS.
exempt in India.
2. The services provided by the
How many of the above statements is/are
government on non-commercial basis
correct?
are exempted from the applicability of
(a) Only one
the GATS.
(b) Only two
3. Market access to trade in services is
(c) All three
provided by the member countries on
(d) None
the ‘negative list approach’.
How many of the above statements is/are
79. Consider the following statements about
correct?
Open Market Sales Scheme (OMSS):
(a) Only one
1. Its primary objective is to sell food
grains to state governments to meet (b) Only two
their Public Distribution Scheme (c) All three
requirements. (d) None
2. It is mandatory for the State
governments to participate in the
82. Consider the following statements:
scheme.
Which of the statements given above is/are Statement-I: India has witnessed an
correct? increase in the production of wheat in the
last decade.
(a) 1 only
(b) 2 only Statement-II: The trends in India’s
cropping pattern indicate an increase in the
(c) Both 1 and 2
area under cultivation for wheat in the last
(d) Neither 1 nor 2
decade.

RAUSIAS-0673 15
https://upscpdf.com/ https://upscpdf.com/
https://t.me/visioniastestseries2024
https://upscmaterial.online/

https://upscpdf.com/ https://upscpdf.com/
Which one of the following is correct in 85. With reference to ‘Claudia Goldin’s U-
respect of the above statements? shaped Curve Hypothesis’, consider the
(a) Both Statement-I and Statement-II are following statements:
correct and Statement-II is the correct 1. The upward trend in the U-shaped
explanation for Statement-I curve is due to a rise in the household
(b) Both Statement-I and Statement-II are incomes.
correct and Statement-II is not the 2. Women participation in the labour force
correct explanation for Statement-I increases with rise in education levels.
(c) Statement-I is correct but Statement-II 3. Gender gap in earnings has narrowed
is incorrect with the rise of the service sector.
(d) Statement-I is incorrect but Statement- How many of the above statements is/are
II is correct correct?
(a) Only one
83. Consider the following statements: (b) Only two
Statement-I: An inverted yield curve points (c) All three
towards a probable economic boom. (d) None
Statement-II: As per an inverted yield
curve, the yield on the short-term bonds is 86. With reference to the National Mission on
higher than the yields of the long-term Edible Oils - Oil Palm (NMEO-OP), consider
bonds. the following statements:
Which one of the following is correct in 1. Its aim is to enhance the edible oilseeds
respect of the above statements? production and oil availability in India
(a) Both Statement-I and Statement-II are by harnessing the oil palm area
correct and Statement-II is the correct expansion and to reduce import burden
explanation for Statement-I on edible oils.
(b) Both Statement-I and Statement-II are 2. One of the targets of the scheme is to
correct and Statement-II is not the increase consumer awareness to
correct explanation for Statement-I maintain the consumption level of the
(c) Statement-I is correct but Statement-II palm oil of 51.00 kg/person/annum till
is incorrect 2025-26.
(d) Statement-I is incorrect but Statement- 3. It offers special assistance of Rs. 70,000
II is correct per hectare to the farmers for planting
material and management for the oil
palm.
84. Which of the following statements does not
apply to the Exchange-Traded Funds How many of the above statements is/are
(ETFs)? correct?

(a) They are a type of pooled investment (a) Only one


securities that operate much like a (b) Only two
mutual fund. (c) All three
(b) They cannot be structured to track the (d) None
price of individual commodities.
(c) They can contain many types of 87. Consider the following statements:
investments, like stocks and bonds. 1. Gross Fixed Capital Formation (GFCF)
(d) Their prices may fluctuate, as they can refers to an increase in the fixed assets
be bought and sold all day. in an economy in a year.

RAUSIAS-0673 16
https://upscpdf.com/ https://upscpdf.com/
https://t.me/visioniastestseries2024
https://upscmaterial.online/

https://upscpdf.com/ https://upscpdf.com/
2. It accounts for the depreciation of the 90. Consider the following statements
fixed capital. regarding the Micro, Small and Medium
3. Financial assets are excluded from the Enterprises Development Act, 2006:
GFCF. 1. The buyers of goods and services
4. Land purchases are excluded from the should mandatorily pay the micro and
GFCF. small enterprises within a maximum
How many of the above statements is/are period of 45 days.
correct about the Gross Fixed Capital 2. In case of further delay in payment, the
Formation (GFCF)? buyer needs to pay compound interest
(a) Only one at three times of the bank rate notified
(b) Only two by the RBI, along with the amount to
(c) Only three the micro and small enterprises.
(d) All four Which of the statements given above is/are
correct?
88. Consider the following statements about (a) 1 only
the National Land Monetization (b) 2 only
Corporation (NLMC): (c) Both 1 and 2
1. It is a wholly-owned Government of (d) Neither 1 nor 2
India company.
2. It carries out monetization of the land
91. The terms ‘MiCA’ and ‘CARF’, seen
and other non-core assets of the
sometimes in the news, are related to
Government agencies and the PSUs.
which of the following?
3. It can lease the surplus land with the
PSUs, but cannot sell them. (a) Cyber security framework.
How many of the above statements is/are (b) Regulation of crypto assets.
correct? (c) Stem cell therapy.
(a) Only one (d) Climate smart agricultural techniques.
(b) Only two
(c) All three 92. Consider the following pairs:
(d) None S.N. Important Reason for being in
location news
89. With reference to Lakhpati Didi scheme,
1. Avdiivka A Ukrainian town
consider the following statements:
captured by Russia.
1. It is an initiative of the Ministry of
Women & Child Development. 2. Cabo Verde An African country
declared malaria-free
2. A Lakhpati Didi is a Self-Help Group
member who earns an annual by the WHO.
household income of Rupees One Lakh 3. Chancay Setting-up of military
or more. base by India.
3. Under this scheme, the women would 4. Nova Establishment of
be trained in skills like plumbing, LED Kakhovka deep-water port by
bulb making and operating and
China.
repairing drones.
How many of the statements given above How many of the above pairs is/are
is/are correct? correctly matched?
(a) Only one (a) Only one
(b) Only two (b) Only two
(c) All three (c) Only three
(d) None (d) All four

RAUSIAS-0673 17
https://upscpdf.com/ https://upscpdf.com/
https://t.me/visioniastestseries2024
https://upscmaterial.online/

https://upscpdf.com/ https://upscpdf.com/
93. With reference to the International Criminal How many of the countries mentioned
Court (ICC), consider the following above are the members of both, ‘BRICS’
statements: and ‘the Shanghai Cooperation
1. It is the principal judicial organ of the Organization (SCO)’?
United Nations for the settlement of
(a) Only four
disputes between the States.
(b) Only five
2. It has no jurisdiction to try the
individuals accused of war crimes or (c) Only six
crimes against humanity. (d) All seven
3. It can only intervene where a State is
unable or unwilling to genuinely carry
96. With reference to the Asia-Pacific Economic
out the investigation and prosecute the
Cooperation (APEC), consider the following
perpetrators.
statements:
How many of the above statements is/are
incorrect? 1. It is an inter-governmental forum of 21
(a) Only one member economies in the Pacific Rim,
(b) Only two that promotes free trade throughout the
(c) All three Asia-Pacific region.

(d) None 2. India is a member of the APEC.


3. Its member countries undertake
94. ‘This water body, mostly inhabited by commitments on voluntary basis, which
microscopic algae and bacteria, is are non-binding in nature.
popularly known as Lac Rose and is How many of the above statements is/are
situated in the Dakar region. It lies in a correct?
depression with its shoreline 6.5 metres
(a) Only one
below the sea level and is separated from
the sea by sand dunes. Despite being a (b) Only two
significant tourist attraction in Dakar, it is (c) All three
experiencing a decline in appeal due to (d) None
various factors, including artisanal salt
mining activities around its vicinity.’
Which water body exhibits the 97. ‘This place is known for the stormy weather
characteristics described in the paragraph and rough seas encountered here. It is at
given above? the convergence of the warm Agulhas
(a) Lake Titicaca Current and the cold Benguela Current.
(b) The Dead Sea Dangerous waves from these currents have
(c) The Senegal’s Pink Lake caused many shipwrecks. According to the
(d) Lake Victoria folklore, these shipwrecks led to the legend
of the Flying Dutchman, a ghost ship
95. Consider the following: doomed to sail the oceans forever after
1. Russia being lost in a severe storm near the place.’
2. Iran The place is:
3. The UAE (a) Kingston
4. India (b) Veracruz
5. China (c) Qingdao
6. Egypt (d) The Cape of Good Hope
7. Saudi Arabia

RAUSIAS-0673 18
https://upscpdf.com/ https://upscpdf.com/
https://t.me/visioniastestseries2024
https://upscmaterial.online/

https://upscpdf.com/ https://upscpdf.com/
98. Consider the following: How many of the above statements is/are
1. The Deccan High-Level Principles correct?
2. The Global Biofuel Alliance (a) Only one
3. The Global Digital Public Infrastructure (b) Only two
Repository (c) All three
4. The Green Development Pact (d) None
How many of the above is/are the
outcome(s) of G20? 100. With reference to the North Atlantic Treaty
(a) Only one Organization (NATO), consider the following
(b) Only two statements:
(c) Only three 1. Any decision to invite a country to join
(d) All four the Alliance is taken by the North
Atlantic Council.
2. Sweden and Ukraine are the members
99. Consider the following statements:
of the NATO.
1. The Black Sea is bordered by Bulgaria
Which of the statements given above is/are
and Romania.
correct?
2. The Danube, the Dniepr and the Ebro
(a) 1 only
rivers discharge into the Black Sea.
(b) 2 only
3. The Black Sea is the world's largest
meromictic basin. (c) Both 1 and 2
(d) Neither 1 nor 2

RAUSIAS-0673 19
https://upscpdf.com/ https://upscpdf.com/
https://t.me/visioniastestseries2024
https://upscmaterial.online/

https://upscpdf.com/ https://upscpdf.com/

https://upscpdf.com/ https://upscpdf.com/
https://t.me/visioniastestseries2024
https://upscmaterial.online/

https://upscpdf.com/ https://upscpdf.com/

https://upscpdf.com/ https://upscpdf.com/
https://t.me/visioniastestseries2024
https://upscmaterial.online/

https://upscpdf.com/ https://upscpdf.com/

https://upscpdf.com/ https://upscpdf.com/
https://t.me/visioniastestseries2024
https://upscmaterial.online/

https://upscpdf.com/ https://upscpdf.com/

Test ID
0673

RAU’S IAS
STUDY CIRCLE
Pre Pareekshan-2024
Mock-2
GS (Paper-1)

Answers
and
Explanations
www.rauias.com | compass.rauias.com
https://upscpdf.com/ https://upscpdf.com/
https://t.me/visioniastestseries2024
https://upscmaterial.online/

https://upscpdf.com/ https://upscpdf.com/

https://upscpdf.com/ https://upscpdf.com/
https://t.me/visioniastestseries2024
https://upscmaterial.online/

https://upscpdf.com/ https://upscpdf.com/
1. Answer: (a) The other important site, where the rock-
Explanation: cut Stupas have been excavated, is
Some paintings have been reported from Anakapalle near Vishakhapatnam.
the Kumaon Hills in Uttarakhand also. In Karnataka, Sannati in Gulbarga district
The rock shelters on the banks of river is the largest Stupa site excavated so far. It
Suyal at Lakhudiyar, about twenty also has a Stupa like the one in Amaravati
kilometres on the Almora – Barechina decorated with sculptural relief.
Road, bear these pre-historic paintings. Sannati Stupa is situated on the banks of
Lakhudiyar literally means ‘one lakh the Bhima river.
caves’. Note: More importantly, a sculpture-
The paintings here can be divided into portrait of Ashoka, seated on his throne,
three categories: Man, animal and with his queens, was also discovered in
geometric patterns in white, black and red Sannati.
ochre. Humans are represented in stick-
like forms. A long-snouted animal, a fox 3. Answer: (b)
and a multiple legged lizard are the main
Explanation:
animal motifs. Wavy lines, rectangle-filled
The style of temple architecture in
geometric designs and groups of dots can
northern India is known as ‘Nagara’. In
also be seen here. One of the interesting
northern India, it is common for an entire
scenes depicted here is of hand-linked
temple to be built on a stone platform, with
dancing human figures. There is some
steps leading up to it.
superimposition of the paintings. The
earliest are in black; over these are red Further, unlike in South India, it does not
ochre paintings and the last group usually have elaborate boundary walls or
comprises white paintings. gateways. While the earliest temples had
just one tower, or Shikhara, the later
temples had several. The Garbhagriha is
2. Answer: (b)
always located directly under the tallest
Explanation: tower.
The remains of a Buddhist monastery, There are many sub-divisions of the
belonging to the 3rd-4th century AD, Nagara temples, depending on the shape of
have been unearthed from Devni Mori, the Shikhara. There are different names for
about two km from Shamlaji (Gujarat). various parts of the temple in different
This site is outside the Gangetic valley. parts of India. However, the most common
Excavation of the Stupa brought to light an name for the simple Shikhara, which is
inscribed casket which contained the relics square at the base and whose walls curve
of Budhha. The inscription clearly or slope inward to a point on the top, is
mentions that the casket contains the called the 'Latina' or the ‘Rekha-Prasada’
bodily relics of Buddha. type of Shikara.
The site provides a visual indication of The second major type of architectural
(early historic) economic and cultural form in the Nagara order is the Phamsana.
connections with Gandhara. Phamsana buildings tend to be broader
Independent Buddha images are found at and shorter than the Latina ones. Their
Amaravati, Nagarjunkonda and Guntapalle roofs are composed of several slabs that
in Andhra Pradesh. gently rise to a single point over the centre
Guntapalle is a rock-cut cave site near of the building, unlike the Latina ones,
Eluru. Small apsidal and circular Chaitya which look like sharply rising tall towers.
halls have been excavated, belonging to the Phamsana roofs do not curve inward,
second century BCE. instead they slope upwards on a straight
incline.

RAUSIAS-0673 1
https://upscpdf.com/ https://upscpdf.com/
https://t.me/visioniastestseries2024
https://upscmaterial.online/

https://upscpdf.com/ https://upscpdf.com/
In many north Indian temples, the 4. Answer: (c)
Phamsana design is used for the Explanation:
Mandapas, while the main Garbhagriha is The Kushanas (c. first century BCE - first
housed in a Latina building. century CE) ruled over a vast kingdom,
Dashavatara Temple, Deogarh (in extending from Central Asia to north-west
Lalitpur district, Uttar Pradesh) was built India. Their history has been reconstructed
in the early sixth century CE. That is, from inscriptions and textual traditions.
about a hundred years or so after the small The notions of kingship they wished to
temples, we just learnt about in Sanchi project are perhaps best evidenced in their
and Udaigiri. This makes it a classic coins and sculpture.
example of a late Gupta period type of Many Kushana rulers also adopted the title
temple. This temple is in the Devaputra, or “the son of the God”,
Panchayatana style of architecture, possibly inspired by the Chinese rulers,
where the main shrine is built on a who called themselves the sons of heaven.
rectangular plinth, with four smaller Colossal statues of the Kushana rulers
subsidiary shrines at the four corners have been found installed in a shrine at
(making it a total number of five shrines, Mat near Mathura (Uttar Pradesh). Similar
hence the name, Panchayatana). statues have been found in a shrine in
The Lakshmana temple of Khajuraho, Afghanistan as well. Some historians feel
dedicated to Vishnu, was built in 954 by this indicates that the Kushanas
the Chandela king, Dhanga. A Nagara considered themselves god-like.
temple, it is placed on a high platform The Kushanas, however, issued the largest
accessed by stairs. hoards of gold coins, first gold coins c. first
The Karkota period of Kashmir is the most century CE. These were virtually identical
significant in terms of architecture. One of in weight with those issued by the
the most important temples is Pandrethan, contemporary Roman emperors and the
built during the eighth and the ninth Parthian rulers of Iran, and have been
centuries. found from several sites in north India and
Central Asia. The widespread use of gold
In keeping with the tradition of a water
coins indicates the enormous value of the
tank attached to the shrine, this temple is
transactions that were taking place.
built on a plinth built in the middle of a
Besides, hoards of Roman coins have been
tank. Although there is evidence of both
found from the archaeological sites in
Hindu and Buddhist followings in
south India. It is obvious that the networks
Kashmir, this temple is a Hindu one,
of trade were not confined within the
possibly dedicated to Shiva.
political boundaries. South India was not a
The architecture of this temple is in part of the Roman empire, but there were
keeping with the age-old Kashmiri tradition close connections through trade.
of wooden buildings. Due to the snowy The Rabatak inscription relates to the rule
conditions in Kashmir, the roof is peaked of the Kushan emperor Kanishka and gives
and slants slowly outward. remarkable clues on the genealogy of the
The temple is moderately ornamented, Kushan dynasty.
moving away from the post-Gupta
aesthetics of heavy carving. A row of 5. Answer: (d)
elephants at the base and a decorated
Explanation:
doorway are the only embellishments on
All Gods mentioned in the question are
the shrine.
mentioned in the Prayaga Prashasti.
Note: The Sun temple at Modhera dates
The Prayaga Prashasti (also known as the
back to the early eleventh century and was
Allahabad Pillar Inscription) was composed
built by Raja Bhimdev I of the Solanki
in Sanskrit by Harishena, the court poet of
dynasty in 1026.

RAUSIAS-0673 2
https://upscpdf.com/ https://upscpdf.com/
https://t.me/visioniastestseries2024
https://upscmaterial.online/

https://upscpdf.com/ https://upscpdf.com/
Samudragupta, the most powerful of the 6. Answer: (d)
Gupta rulers (c. fourth century CE). Explanation:
In praise of Samudragupta – This is an The Periplus of the Erythraean Sea:
excerpt from the Prayaga Prashasti: He "Periplus" is a Greek word meaning sailing
was without an antagonist on earth; he, by
around and "Erythraean" was the Greek
the overflowing of the multitude of (his)
name for the Red Sea.
many good qualities adorned by hundreds
of good actions, has wiped off the fame of
other kings with the soles of (his) feet; (he 7. Answer: (c)
is) Purusha (the Supreme Being), being the Explanation:
cause of the prosperity of the good and the William Finch: He was a European
destruction of the bad (he is)
traveller, who arrived in India in 1608. He
incomprehensible; (he is) one whose tender
wrote about his Ayodhya visit. Finch
heart can be captured only by devotion and
visited Ayodhya between 1608 and 1611,
humility; (he is) possessed of compassion;
(he is) the giver of many hundred- when Jahangir ruled the Mughal empire.
thousands of cows; (his) mind has received He called Ayodhya a “city of ancient note”.
ceremonial initiation for the uplift of the Joseph Tieffenthaler: In 1740, during the
miserable, the poor, the forlorn and the reign of the later Mughals, more than three
suffering; (he is) resplendent and embodied decades after Aurangzeb’s death, Jesuit
kindness to mankind; (he is) equal to (the missionary Joseph Tieffenthaler visited
gods) Kubera (the god of wealth), Varuna India. He wrote “Avad, called as Adjudea by
(the god of the ocean), Indra (the god of educated Hindus, is a city of very olden
rains) and Yama (the god of death)… times.”
Joannes de Laet: Joannes de Laet became
a director of the Dutch East India
Company in the 1620s. He says, “Not far
from this city (Ayodhya) may be seen the
ruins of the fort and palace of Ramachand,
whom the Indians regard as God most
high; they say that he took human flesh so
that he might see the great Tamasha of the
world.”

8. Answer: (b)
Explanation:
Vijayanagara: The first dynasty, known as
the Sangama dynasty, exercised control till
1485. They were supplanted by the
Saluvas, the military commanders, who
remained in power till 1503, when they
were replaced by the Tuluvas.
Krishnadeva Raya (ruled 1509-29)
belonged to the Tuluva dynasty.
Krishnadeva Raya’s rule was characterized
by expansion and consolidation. This was
the time when the land between the
Tungabhadra and Krishna rivers (the
Raichur doab) was acquired (1512), the

RAUSIAS-0673 3
https://upscpdf.com/ https://upscpdf.com/
https://t.me/visioniastestseries2024
https://upscmaterial.online/

https://upscpdf.com/ https://upscpdf.com/
rulers of Orissa were subdued (1514) and they could not hope to enjoy the support of
severe defeats were inflicted on the Sultan the citizens, and the army stationed there.
of Bijapur (1520). Although the kingdom But Alauddin had some additional reasons
remained in a constant state of military for controlling the market. The Mongol
preparedness, it flourished under invasions of Delhi had pin-pointed the
conditions of unparalleled peace and need to raise a large army to check them.
prosperity. But such an army would soon exhaust his
Krishnadeva Raya is accredited with treasures unless he could lower the prices,
building some fine temples and adding and hence, lower their salaries.
impressive Gopurams to many important To realize his objectives, Alauddin
south Indian temples. He also founded a proceeded in a characteristically thorough
suburban township near Vijayanagara, way.
called Nagalapuram, after his mother. In order to ensure a regular supply of
Some of the most detailed descriptions of cheap food grains, he declared that the
Vijayanagara come from his time or just land revenue in the doab region, that is,
after. the area extending from Meerut near the
He composed a work on statecraft in Yamuna to the border of Kara near
Telugu, known as the Amuktamalyada. Allahabad, would be paid directly to the
state, i.e., the villages in the area would not
9. Answer: (d) be assigned in Iqta to anyone. Further, the
Explanation: land revenue was raised to half of the
produce.
For the contemporaries, Alauddin’s
measures to control the markets was one The merchants were commandeered to
of the great wonders of the world. In a transport grains from villages to Delhi.
series of orders after his return from the Control of the prices of horses was
Chittor campaign, Alauddin sought to fix important for the Sultan, because without
the cost of all commodities, from food the supply of good horses at reasonable
grains, sugar and cooking oil, to a needle, prices to the army, the efficiency of the
and from costly imported cloth to horses, army could not be maintained. The
cattle, and slave boys and girls. position of the supply of horses had
For this purpose, he set up three markets improved as a result of the conquest of
at Delhi—one market for food grains, the Gujarat. Good quality horses could be sold
second one for costly cloth and the third only to the state.
one for horses, slaves and cattle.
Each market was under the control of a 10. Answer: (a)
high officer, called Shahna Explanation:
(Superintendent of the market), who The Diwan-i-Rasalat was under a foreign
maintained a register of the merchants, minister. His duty was to keep himself
and strictly controlled the shopkeepers in touch with the ambassadors and the
and the prices. envoys. He also dealt with diplomatic
He was given a Barid and other officers correspondence. The work of charity and
to help him in his work. endowment was also in his hands.
The regulation of prices, especially food The Minister-in-charge of Diwan-i-Insha
grains, was a constant concern of the had to draft the royal proclamation and
medieval rulers, because without the dispatches. He was in-charge of the
supply of cheap food grains to the towns, government records. His duty was also

RAUSIAS-0673 4
https://upscpdf.com/ https://upscpdf.com/
https://t.me/visioniastestseries2024
https://upscmaterial.online/

https://upscpdf.com/ https://upscpdf.com/
to correspond with the Governors and In the first stage, the Company undertook
other local officers. to lend its troops to a friendly Indian prince
The Sarais were used as Dak Chaukis. to assist him in his wars, as for instance,
the Treaty with the Nizam (1768).
The Pargana was the main unit for revenue
collection. It consisted of a large number of In the second stage, the Company's troops
villages. The Shiqdar was responsible for took the field on their own account with
the assistance of an Indian ally who made
the collection of land revenue.
common cause with them.
Sher Shah introduced a new coin, called
The next stage was reached when the
Dam. He also abolished the old and mixed
Indian ally was not to supply men, but
metal currency. The names on the coins
money. The Company undertook to raise,
were given in Devanagari script. Gold coins
train and equip an army under the English
were also introduced. The ratio of exchange
officers and render available to the ally a
between the Dam and the rupee was fixed
fixed number of troops on receiving a sum
at 64 to 1. of money towards the cost of these troops,
as for instance in the Treaty with
11. Answer: (b) Hyderabad (1798).
Explanation: The final stage was the next logical step.
The term, which the Indo-Persian sources The Company undertook to defend the
of the Mughal period most frequently used territories of an Indian ally and for that
to denote a peasant, was Raiyat (plural, purpose stationed a subsidiary force in the
territory of the state. The Indian ally was
Riaya) or Muzarian.
asked not to pay money but surrender
In addition, we also encounter the terms
territory, from the revenues of which the
Kisan or Asami. The sources of the
expenses of the subsidiary force were to be
seventeenth century refer to two kinds
met, as for instance the Treaty with the
of peasants – Khud-Kashta and Pahi- Nizam (1800).
Kashta.
Wellesley's special contribution was that he
The former were the residents of the greatly developed and elaborated the
village in which they held their lands. system and applied it in the case of almost
The latter were non-resident every Indian state. The earliest subsidiary
cultivators, who belonged to some other treaty negotiated by the Company was with
village, but cultivated lands elsewhere the Nawab of Oudh in 1765, by which the
on a contractual basis. Company undertook to defend the frontiers
People became Pahi-Kashta either out of of Oudh on the condition of the Nawab
choice – for example, when the terms of defraying the expenses of such defence. A
revenue in a distant village were more British Resident was also stationed at
favourable – or out of compulsion – for Lucknow.
example, forced by economic distress after The first time the Company insisted that
a famine. the subsidiary state should have no foreign
relations was in the treaty with the Nawab
of Carnatic concluded by Cornwallis in
12. Answer: (c) 1787.
Explanation: Later, Sir John Shore (The then Governor-
Alfred Lyall (A British civil servant and General of Bengal) in the treaty with the
literary historian) mentions four stages Nawab of Oudh (21st January, 1798)
in the Company's participation in Indian insisted that the Nawab was not to hold
wars: communications with or admit into his

RAUSIAS-0673 5
https://upscpdf.com/ https://upscpdf.com/
https://t.me/visioniastestseries2024
https://upscmaterial.online/

https://upscpdf.com/ https://upscpdf.com/
service other European nationals. The Both, the Reformist Movements and the
demand for surrender of territory in Revivalist Movements depended on a
commutation of cash money was the next varying degree on an appeal to the lost
logical step. Usually the monetary purity of the religion they sought to reform.
demands of the Company for the upkeep of The only difference between one Reform
the subsidiary force were so heavy that the Movement and the other lay in the degree
Indian states, whose revenues shrank or to which it relied on tradition, or on reason
expanded according to the monsoon, were and conscience.
not in a position to meet those and usually
Another significant aspect of all the Reform
fell in arrears.
Movements was their emphasis on both
A typical Subsidiary Treaty was
religious reforms and social reforms. This
negotiated on the following terms and
link was primarily due to two main
conditions:
reasons:
1. The Indian state was to surrender its
(a) Almost every social custom and
external relations to the care of the
institution in India derived sustenance
Company and was to make no wars. It
was to conduct negotiations with other from religious injunctions and
states through the Company; sanctions. This meant that no social
reform could be undertaken unless the
2. A bigger state was to maintain an army
within its territories commanded by the existing religious notions, which
British officers for the 'preservation of sustained the social customs, were also
public peace' and the ruler was to cede reformed,
territory in full sovereignty for the (b) The Indian reformers well understood
upkeep of that force; the close interrelation between different
3. A smaller state was required to pay aspects of human activities.
tribute in cash to the Company; Rammohun Roy, for example, believed
4. The state was to accept a British that religious reform must precede
Resident at its head-quarters; demand for social reform or political
5. The state was not to employ Europeans rights.
in its service, without the consultation Rammohan Roy accepted the concept of
of the Company; one God, as propounded by the
6. The Company was not to interfere in Upanishads. For him, the God was
the internal affairs of the state; and shapeless, invisible, omnipresent and
7. The Company was to protect the Indian omnipotent, but the guiding spirit of the
state against foreign enemies of 'every universe and omniscient.
sort or kind'. In August, 1828, Roy founded the Brahmo
Sabha, which was later renamed the
13. Answer: (a) Brahmo Samaj. The Trust Deed executed
Explanation: in 1830 explained the object of the Brahmo
Samaj as "the worship and adoration of the
The ‘Reform Movements’ fall in two
Eternal, Unsearchable, Immutable being
broad categories:
who is the Author and Preserver of the
One, the Reformist Movements, like the
Universe".
Brahmo Samaj, the Prarthana Samaj and
The Samaj declared its opposition to idol
the Aligarh Movement;
worship and "no graven image, statue or
Two, the Revivalist Movements, like the
sculpture, carving, painting, picture,
Arya Samaj, the Ramakrishna Mission and
portrait or the likeness of anything was to
the Deoband Movement.
be allowed in the Samaj building. There
was no place for priesthood in the Samaj

RAUSIAS-0673 6
https://upscpdf.com/ https://upscpdf.com/
https://t.me/visioniastestseries2024
https://upscmaterial.online/

https://upscpdf.com/ https://upscpdf.com/
building. There was no place for priesthood published in his famous work Satyartha
in the Samaj nor sacrifices of any kind Prakash (The True Exposition).
were allowed. The worship was performed Dayanand launched a frontal attack on
through prayers and meditation and numerous abuses (like idolatry,
readings from the Upanishads.” polytheism, belief in magic, charms,
From the beginning, the appeal of the animal sacrifies, feeding the dead through
Brahmo Samaj had remained limited to the Sraddhas etc.), that had crept into the
intellectuals and the educationally Hindu religion in the 19th century.
enlightened Bengalis living in the towns. The Swami asserted every Hindu's right to
read and interpret the Vedas. He strongly
The orthodox Hindus, led by Raja
condemned the caste system based on
Radhakant Deb, organized the Dharma
birth, though he subscribed to the Vedic
Sabha with the object of countering the
notion of the four-Varna system in
propaganda of the Brahmo Samaj.
which a person was not born in any
The early death of Rammohan, in 1833, left Varna (caste), but was identified as a
the Brahmo Samaj without the guiding Brahmin, Kshatriya, Vaishya or Shudra
soul and a steady decline set in. according to the occupation the person
The Arya Samaj Movement was an outcome followed.
of reaction to Western influences. It was
revivalist in form, though not in content. 14. Answer: (a)
The founder, Swami Dayanand, rejected Explanation:
Western ideas and sought to revive the
The Battle of Plassey (1757) was
ancient religion of the Aryans. followed, in the words of the Bengali
For fifteen years (1845-60), Dayanand poet Nabin Chandra Sen, by “A Night of
wandered as an ascetic in the whole of Eternal Gloom for India”.
India studying Yoga. In 1875, he formally After the Battle of Plassey, the East India
organized the first Arya Samaj unit at Company was granted undisputed right to
Bombay (Mumbai). A few years later, the free trade in Bengal, Bihar and Orissa.
headquarters of the Arya Samaj were In addition, the Nawab paid large sums as
established at Lahore. For the rest of his ‘gifts’, or bribes, to the high officials of the
life, Dayanand extensively toured India for Company. Clive later estimated that the
the propagation of his ideas. Company and its servants had collected
Dayanand's ideal was to unite India more than 30 million rupees from the
religiously, socially and nationally— Aryan puppet Nawab. It was also understood that
religion to be the common religion of all, a the British merchants and the officials
classless and casteless society, and an would no longer be asked to pay any taxes
India free from foreign rule. He looked on on their private trade.
the Vedas as India's 'Rock of Ages', the true
original seed of Hinduism. His motto was 15. Answer: (c)
'Go back to the Vedas'. Explanation:
He gave his own interpretation of the The Nehru Report (1928) visualized a
Vedas. He disregarded the authority of the ‘Parliamentary System’ with full
later Hindu scriptures, like the Puranas, responsible government and joint
and described them as the work of the electorates with time-bound reservation of
lesser men and responsible for the evil seats for the minorities.
practices of idol worship and other
The Report laid emphasis on securing
superstitious beliefs in Hindu religion.
fundamental human rights for the people
Dayanand condemned idol worship and
of India. These included the right to the
preached unity of Godhead. His views were
freedom of conscience and the free
RAUSIAS-0673 7
https://upscpdf.com/ https://upscpdf.com/
https://t.me/visioniastestseries2024
https://upscmaterial.online/

https://upscpdf.com/ https://upscpdf.com/
profession and practice of religion, the in April, 1919, the Anti-Rowlatt Satyagraha
right to assemble peacefully and without lost energy.
arms and to form associations or unions,
equal rights for men and women and the 17. Answer: (a)
right to free elementary education.
Explanation:
Interestingly, the secular character of the
The Swatantra Party, the first authentic
State was listed as a Fundamental Right.
all-India secular conservative party, came
It also recommended that the distribution into being in 1959. It had a number of
of the provinces should take place on a distinguished leaders, most of them old
linguistic basis. Congressmen, like C. Rajagopalachari,
Minoo Masani, N.G. Ranga and K.M.
16. Answer: (c) Munshi.
Explanation: Right-wing groups and parties had earlier
After the Mont-Ford reforms, some leaders, existed at the local and regional levels, but
like Mohammad Ali Jinnah, Annie Besant, Swatantra’s formation was the first attempt
G.S. Kharpade and B.C. Pal, left the to bring these highly fragmented right-wing
Congress, as they believed in a forces together under the umbrella of a
constitutional and lawful struggle, while single party. The provocation was the left
some others, like Surendranath turn which the Congress took at Avadi and
Banerjea, founded the Indian National the Nagpur Resolutions.
Liberal Federation (1919), and played a The Party stood for free, private
minor role in the national politics enterprise and opposed the active role
henceforward. of the State in economic development. It
The Banaras Hindu University is a wanted to radically restrict centralized
collegiate, central and research planning and the role of the public sector,
university located in Varanasi (Uttar as also state regulation of the economy. It
Pradesh) and it was founded in 1916. opposed any nationalization of private
The University was established by Madan enterprise and any extension of land
Mohan Malaviya, with the financial reforms, especially fixation of ceilings on
support of the Maharaja of Dharbhanga, land holdings.
Rameshwar Singh, the Maharaja of In foreign affairs, the Swatantra Party
Banaras, Prabhu Narayan Singh, and opposed non-alignment and a close
lawyer Sunder Lal. relationship with the Soviet Union, and
In the Nagpur session of Congress (1920), advocated an intimate connection with the
a decision was taken to create the All- United States and Western Europe.
India Tilak Memorial Swaraj Fund (in the
memory of Bal Gangadhar Tilak) to raise 18. Answer: (b)
funds for the Non-Cooperation Movement. Explanation:
M. K. Gandhi began a crusade against Recently, the Union Ministry of
the Rowlatt Bill and set-up the Environment, Forest and Climate Change
Satyagraha Sabha in 1919, at Bombay
has notified the Captive Elephant (Transfer
(Mumbai). During this tumult, M.K.
or Transport) Rules, 2024.
Gandhi gave the well-known expression, "It
The Rules lay down the procedure to be
is my firm conviction that we will get
followed for the transfer of the captive
salvation just through torment and not by
changes dropping on us from the English elephants within a state or between two
they utilize beast, we soul power". After the states. It authorizes the Chief Wildlife
episode of the Jallianwala Bagh slaughter Warden of the states and the UTs to permit

RAUSIAS-0673 8
https://upscpdf.com/ https://upscpdf.com/
https://t.me/visioniastestseries2024
https://upscmaterial.online/

https://upscpdf.com/ https://upscpdf.com/
or reject the transfer of the captive of India and non-profit organizations, in
elephants. January, 2024. There are 718 snow
The captive elephants can be transported leopards in India.
to other states after a thorough fitness • According to the report, only 34 per
examination. A veterinary expert should cent of the 1,20,000 square kilometre
examine the elephant and issue a fitness snow leopard habitat in India is legally
certificate. The owner of the elephant protected. Almost 70 per cent of it,
should file an application, along with the crucial for the predator, remains
veterinary expert’s report, to the Chief unprotected.
Wildlife Warden of the state to which the
• The report emphasized the need for a
elephant is transported.
dedicated ‘Snow Leopard Cell’ at the
The Chief Wildlife Warden, within seven
Dehradun-based Wildlife Institute of
days, should forward the application to an
India (WII), under the Union Ministry of
officer, not less than the rank of the deputy
Environment, Forest and Climate
conservator of forest.
Change.
The transfer will be allowed only under
• According to the report, the highest
the following three circumstances:
number of snow leopards (listed as
(i) The first is that the owner should no
‘Vulnerable’ in the International
longer be in a position to maintain the
Union for Conservation of Nature
elephant.
Red List) is in Ladakh (477). It is
(ii) The transfer can be done if the elephant
followed by Uttarakhand (124
can get a better upkeep, compared to
animals) and Himachal Pradesh (51).
the present circumstances.
Sikkim and Jammu & Kashmir
(iii) The third condition is that the Chief
recorded 21 and nine individuals,
Wildlife Warden should deem it fit and
respectively.
proper in the circumstances of the
case, considering the better upkeep of o The Hemis National Park, famous
the elephant. for snow leopards, is in Ladakh.
The transfer of the captive elephants will o The snow leopard’s wide nose
not be permissible unless the genetic warms cold air before it enters its
profile of the animal has been entered in lungs.
the electronic monitoring application of the o Snow leopards can mew, growl, and
Ministry of Environment, Forest and puff … but they cannot roar.
Climate Change (MoEFCC). o Adult snow leopards are solitary,
“These Rules [The ‘Captive Elephant but young siblings will cuddle and
(Transfer or Transport) Rules, 2024], play.
when read alongside the 2022
o Its scientific name is Panthera
Amendment to the Wildlife (Protection)
uncia.
Act of 1972, which granted exemptions
for the transfer of the captive elephants
for ‘religious and other purposes,’ might 20. Answer: (c)
be susceptible to misuse, particularly in Explanation:
the trade of the elephants.
Brumation is the term to describe a period
of dormancy, or slowed activity in the
19. Answer: (b) reptiles. It typically occurs during the
Explanation: colder months, when the temperatures
Recently, a report (Status Report of Snow drop, and the food becomes scarce.
Leopards in India), related to the ‘snow
leopards’, was released by the Government

RAUSIAS-0673 9
https://upscpdf.com/ https://upscpdf.com/
https://t.me/visioniastestseries2024
https://upscmaterial.online/

https://upscpdf.com/ https://upscpdf.com/
Reptiles enter a state of dormancy to by 2035, aligning with the broader
conserve energy and survive these adverse objective of minimizing the risks associated
environmental conditions. with these substances.
During brumation, the reptiles may retreat The Framework highlights an integrated
to underground burrows, rock crevices or financing approach, including
other sheltered areas, where the mainstreaming, private sector financing,
and external financing through a
temperatures are relatively more stable.
dedicated Global Framework on
Their metabolism slows significantly,
Chemicals Fund, managed by the United
allowing them to go weeks, or even
Nations Environment Programme (UNEP).
months, without eating. This period of
The Global Environment Facility (GEF) is
reduced activity allows the reptiles to
urged to increase funding for the
conserve energy and minimize their
Framework.
resource requirements.
A significant feature of the new
The researchers have observed instances of Framework, like its predecessor, the
brumation in various reptilian species Strategic Approach to International
across habitats. Chemicals Management (SAICM), is its
Such species include the box turtles and inclusive, multi-stakeholder and multi-
the painted turtles, which burrow into the sectoral approach.
mud at the bottom of ponds or lakes. It aims to ensure representation from
Snakes may seek refuge in underground various sectors, including governments,
dens or caves, while lizards may hide international organizations and
representatives of industry and public
under rocks or within vegetation.
interest groups, as well as representatives
Alligators also go through brumation.
from areas, such as environment, health,
Note: Hibernation is an involuntary agriculture and labour.
biological process for some endotherms All stakeholders participate on equal
(warm-blooded animals), where the footing in both negotiations and
animal’s body temperature, heart rate, implementation processes.
breathing and other metabolic processes Note: The Bonn Declaration: A high-level
slow significantly. Bats go through declaration—the ‘Bonn Declaration’—
hibernation. recognizes that chemical pollution
While hibernation is almost exclusively contributes to millions of deaths, illnesses
seen in the mammals, the and disabilities each year.
‘ectotherms’, or the cold-blooded animals, It emphasizes that effective management of
including fish, reptiles and amphibians, chemicals and waste will significantly
contribute to achieving the objectives of
cannot hibernate. The dormancy they
‘the Paris Agreement on Climate’, and the
exhibit is called brumation.
goals and targets of ‘the Kunming-Montreal
Global Biodiversity Framework’.
21. Answer: (a) The Declaration commits to preventing
Explanation: exposure to harmful chemicals, phasing
The Global Framework on Chemicals out the most dangerous substances, and
emphasizes the importance of supporting protecting and respecting human rights for
the implementation of the national legal the well-being of the present and future
frameworks. Additionally, it includes a generations.
specific target related to the phase-out of
highly hazardous pesticides in agriculture

RAUSIAS-0673 10
https://upscpdf.com/ https://upscpdf.com/
https://t.me/visioniastestseries2024
https://upscmaterial.online/

https://upscpdf.com/ https://upscpdf.com/
22. Answer: (a) regulations and policies for trees outside
Explanation: forests (TOF); increase access to finance,
The ‘Trees Outside Forests in India’ (TOFI) quality planting materials and value-chain
is a five-year programme that aims to support; bridge gaps in technology and
significantly expand the area under ‘Trees market info through data, monitoring and
Outside Forests’, while bringing inclusive decision tools.
environmental and livelihood benefits. It is The TOFI’s overall goal is to significantly
a joint programme of the United States expand the area under TOF for enhancing
Agency for International Development both livelihoods and ecosystem services,
(USAID) and the Ministry of Environment, especially carbon sequestration.
Forest and Climate Change of the Implemented by a consortium led by the
Government of India, being implemented Centre for International Forestry Research
by a consortium of nine organizations, led (CIFOR) and World Agroforestry (ICRAF),
by CIFOR-ICRAF (The Centre for the programme will be implemented in
International Forestry Research and World seven states, including Andhra Pradesh,
Agroforestry). Assam, Haryana, Odisha, Rajasthan,
This programme seeks to strengthen the Tamil Nadu and Uttar Pradesh.
enabling environments to improve

23. Answer: (b)


Explanation:

RAUSIAS-0673 11
https://upscpdf.com/ https://upscpdf.com/
https://t.me/visioniastestseries2024
https://upscmaterial.online/

https://upscpdf.com/ https://upscpdf.com/

The Sea Slug


Note: Recently, the Zoological Survey of India (ZSI) named a new marine species of head-shield
‘sea slug’ with ruby red spot.
This species, belonging to the Melanochlamys genus, was discovered from Digha of West Bengal
coast and Udaipur of Odisha coast.
The new species of head-shield sea slug, which is found nowhere in the world, is named
as Melanochlamys droupadi.
It is a small invertebrate with a maximum length of upto 7 mm, brownish black in colour,
with a ruby red spot in the hind end, shell inside the body, hermaphrodite, normally crawling
on the intertidal zone, which left the crawl mark behind them in the sandy beaches.
The species of the genus Melanochlamys are characterised morphologically by a short, blunt
and cylindrical body, and a smooth dorsal surface with two dorsal equal or unequal shields,
named the ‘anterior cephalic’ and the ‘posterior shield’.
According to the ZSI, the species of this group are generally distributed in the temperate regions
of the Indo-Pacific Oceanic realm, but three species are truly tropical
distributed, Melanochlamys papillata from the Gulf of Thailand; Melanochlamys
bengalensis from West Bengal and Odisha coast; and the present species.

24. Answer: (b) the combined diversity at all the levels of


Explanation: the biological organization.
In our biosphere, immense diversity (or The most important of them are:
heterogeneity) exists, not only at the (i) Genetic diversity: A single species
species level, but at all levels of biological might show high diversity at the
organization, ranging from macro- genetic level over its distributional
molecules within the cells to biomes. range. The genetic variation shown by
Biodiversity is the term popularized by the the medicinal plant, Rauwolfia
socio-biologist Edward Wilson to describe vomitoria, growing in different
Himalayan ranges, might be in terms

RAUSIAS-0673 12
https://upscpdf.com/ https://upscpdf.com/
https://t.me/visioniastestseries2024
https://upscmaterial.online/

https://upscpdf.com/ https://upscpdf.com/
of the potency and concentration of Three of these Sites, the Ankasamudra
the active chemical (reserpine) that the Bird Conservation Reserve; the
plant produces. India has more than Aghanashini Estuary; and the Magadi
50,000 genetically different strains of Kere Conservation Reserve are located
rice and 1,000 varieties of mango. in Karnataka, whereas two, the
(ii) Species diversity: The diversity at Karaivetti Bird Sanctuary; and the
the species level, for example, the Longwood Shola Reserve Forest are in
Western Ghats, has a greater Tamil Nadu.
amphibian species diversity than With the addition of these five wetlands to
the Eastern Ghats. the List of Wetlands of International
(iii) Ecological diversity: At the Importance, the total area covered under
ecosystem level, India, for instance, the Ramsar Sites is now 1.33 million ha,
with its deserts, rain forests, which is an increase of 5,523.87 ha from
mangroves, coral reefs, wetlands, the existing area of 1.327 million ha.
estuaries and alpine meadows, has a Tamil Nadu continues to have the
greater ecosystem diversity than a maximum number of Ramsar Sites (16
Scandinavian country, like Norway. sites), followed by Uttar Pradesh (10 sites).
Species diversity on the Earth is not
uniformly distributed, but shows 26. Answer: (b)
interesting patterns. It is generally highest Explanation:
in the tropics and decreases towards the
The Carbon Border Adjustment Mechanism
poles.
(CBAM) is the European Union (EU)'s
Important explanations for the species landmark tool to fight carbon leakage and
richness of the tropics: The tropics had one of the central pillars of the EU's
more evolutionary time; they provide a ambitious ‘Fit for 55 Agenda’. It will
relatively constant environment; and they equalize the price of carbon between the
receive more solar energy which domestic products and imports. This will
contributes to greater productivity. ensure that the EU's climate policies are
Species richness is also the function of the not undermined by production re-locating
area of a region; the species-area to the countries with less ambitious green
relationship is generally a rectangular standards, or by the replacement of the EU
hyperbolic function. products by more carbon-intensive
imports.
25. Answer: (a) It is a measure that encourages the global
Explanation: industry to embrace greener and more
The World Wetlands Day (WWD) is sustainable technologies.
observed on 2nd February every year In its transitional phase, the CBAM will
worldwide to commemorate the signing of only apply to the imports of cement, iron
the Ramsar Convention on Wetlands of and steel, aluminium, fertilizers, electricity
International Importance, in 1971. and hydrogen. EU importers of those
India is a party to the Convention since goods will have to report on the volume of
1982 and on the eve of the WWD-2024, their imports and the greenhouse gas
India has increased its tally of Ramsar (GHG) emissions embedded during their
Sites (The Wetlands of International production, but without paying any
Importance) to 80 by designating five more financial adjustment at this stage.
wetlands as Ramsar Sites. India is reportedly among the top eight
countries that will be adversely affected by
the CBAM. As per the Global Trade
RAUSIAS-0673 13
https://upscpdf.com/ https://upscpdf.com/
https://t.me/visioniastestseries2024
https://upscmaterial.online/

https://upscpdf.com/ https://upscpdf.com/
Research Initiative Report, in 2022, 27% of Second, the EU could collect the tax and
India’s exports of iron, steel and return the funds to such countries to
aluminium products, worth $8.2 billion, invest in their green technologies. This
went to the EU. It is estimated that a few of appears pragmatic, especially as the CBAM
its core sectors, such as steel, will be will enter the definitive phase in 2026.
greatly affected by the CBAM. Thus, the ongoing negotiations with the EU
India seems to have limited options to must be closely observed for this purpose.
navigate the CBAM framework. The first India has already challenged the CBAM
would be to challenge the practice as being before the World Trade Organization under
violative of the ‘Common but Differentiated the ‘Special and Differential Treatment
Responsibilities Principle’, agreed upon Provisions’.
under the Paris Agreement.

27. Answer: (d)


Explanation:
About the Montecincla and the Sholicola: These birds are in the news due to the Humboldt’s
enigma. These are the songbirds in the Western Ghats.
The Montecincla and the Sholicola

Alexander von Humboldt (1769-1859) was a polymath who recorded observations on various
natural phenomena.
He suggested that there was a relationship between temperature, altitude and humidity on one
hand, and the occurrence patterns of the species – or their biodiversity – on the other.
His example of choice was the Chimborazo mountain in Ecuador, which has today become an
important illustration of mountain diversity.
Two centuries later, a group of bio-geographers – the scientists who explore the relationship of
diversity with geography – used modern tools to take another look at the drivers of biodiversity.
Based on their findings, they proposed their own version of the link between biodiversity and
mountains, and called it the Humboldt’s enigma.
It is known that the mountains host two processes that generate biodiversity:
First: Geological processes, like uplifts, result in new habitats where new species arise, so the
habitats are ‘cradles’.
Second: Species on some climatologically stable mountains persist there for a long time, so
these spots are ‘museums’ that accumulate many such species over time.

RAUSIAS-0673 14
https://upscpdf.com/ https://upscpdf.com/
https://t.me/visioniastestseries2024
https://upscmaterial.online/

https://upscpdf.com/ https://upscpdf.com/
The coastal tropical sky islands (The mountains surrounded by the lowlands), like the Shola Sky
Islands in the Western Ghats, are a good example. Here, old lineages have persisted on the
mountain tops as climates and habitats fluctuated around them in the lower elevations.
This is the reason that some of the oldest bird species in the Western Ghats, such as
the Montecincla and the Sholicola, are housed on the Shola Sky Islands.
Sometimes, the same mountain can be both a cradle for some species and museum for others,
depending on the species’ ecologies.

28. Answer: (a) and anaerobiosis inhibit decomposition


Explanation: resulting in the build-up of organic
The decomposers break down complex materials.
organic matter into inorganic substances,
like carbon dioxide, water and nutrients, 29. Answer: (c)
and the process is called decomposition. Explanation:
Dead plant remains, such as leaves, bark An ‘elephant corridor’ can be described as
and flowers, and dead remains of animals, a strip of land that enables elephant
including fecal matter, constitute detritus, movement between two or more friendly
which is the raw material for habitats. West Bengal has the maximum
decomposition. The important steps in the number of elephant corridors (26),
process of decomposition are amounting to 17 per cent of the total
fragmentation, leaching, catabolism, corridors. The total number of elephant
humification and mineralization. corridors has increased to 150, a
Detritivores (e.g., earthworms) break down remarkable increase from the 88 registered
detritus into smaller particles. This process in 2010.
is called ‘fragmentation’. By the process of India has the largest number of Asian
leaching, water-soluble inorganic nutrients elephants. The elephant population in the
go down into the soil horizon and get country is estimated to be over 30,000 and
precipitated as unavailable salts. Karnataka has the highest elephant
Bacterial and fungal enzymes degrade population in India.
detritus into simpler inorganic substances. The Sigur Elephant Corridor is the only
This process is called ‘catabolism’. one of its kind connecting the Western
Decomposition is largely an oxygen- Ghats and the Eastern Ghats, and
requiring process. The rate of facilitating easy movement of the Asian
decomposition is controlled by chemical elephants from Kerala, Tamil Nadu and
composition of detritus and climatic Karnataka, or in the Nilgiri Biosphere
factors. In a particular climatic Reserve.
condition, the decomposition rate is
slower, if detritus is rich in lignin and 30. Answer: (b)
chitin, and quicker, if detritus is rich in
Explanation:
nitrogen and water-soluble substances,
• Seagrasses are different from seaweeds,
like sugars. Temperature and soil
and are found in salty and brackish
moisture are the most important climatic
waters at a shallow depth.
factors that regulate decomposition
through their effects on the activities of soil • They grow both horizontally and
microbes. vertically.
Warm and moist environment favours • They are called the foundational plant
decomposition, whereas low temperature species or the ecosystem engineers.

RAUSIAS-0673 15
https://upscpdf.com/ https://upscpdf.com/
https://t.me/visioniastestseries2024
https://upscmaterial.online/

https://upscpdf.com/ https://upscpdf.com/
• They are also called the “Lungs of the sides of a shallowly rooted tree. Typically,
Sea”. they are found in the nutrient-poor tropical
• Seagrasses are found all across the forest soils that may not be very deep. They
Indian coast, along with the Andaman prevent the tree from falling over (hence
& Nicobar and the Lakshwadeep the name buttress), while also gathering
islands. more nutrients.
• The largest seagrass bed in the Nutrient uptake: Pneumatophores (aerial
country is in the Gulf of Mannar and roots) allow the mangroves to absorb gases
the Palk Bay. directly from the atmosphere.
• They are used for fertilizing fields, Country-wise: About 40% of global
insulating houses, thatch roofs, etc. mangrove cover is in just four countries:
Indonesia > Brazil > Nigeria > Mexico.
31. Answer: (c)
Explanation: 33. Answer: (b)
The ‘community forest resource rights’, Explanation:
along with the ‘community rights’ (CRs), The Dokriani Glacier is in the Garhwal
under Sections 3(1)(b) and 3(1)(c) of the Himalayas.
Scheduled Tribes and Other Traditional The Drang-Drung Glacier is retreating and
Forest Dwellers (Recognition of Forest located in the Zanskar Range.
Rights) Act, 2006 (The FRA, 2006), which
The Milam Glacier is situated in the
include the ‘Nistar’ rights and the rights
Kumaon Himalayas, originating from the
over non-timber forest products, ensure
eastern slopes of the Trishuli. The river
sustainable livelihoods of the community.
Gori Ganga has its source here.
These rights give the authority to the Gram
The Pensilungpa Glacier, located in the
Sabha to adopt the local traditional
Zanskar Range, is retreating, and a recent
practices of forest conservation and
study has attributed the retreat to an
management within the community forest
increase in the temperature and decrease
resource boundary.
in precipitation during winters.
Note: Recently, the World Meteorological
32. Answer: (b)
Organization (WMO) made an urgent call to
Explanation: action over the melting cryosphere.
Mangroves are salt-tolerant plant
communities, or halophytes, found in the
34. Answer:(a)
inter-tidal zones in the tropical and sub-
Explanation:
tropical zones of the world. Mangroves are
critical for storing blue carbon and the • Recently, the Supreme Court froze
protection of the coastal ecosystems. the Assam Government’s move to
withdraw a nearly 26-year-old
Pneumatophores, commonly found in the
notification constituting the
mangrove species that grow in saline mud
Pobitora Wildlife Sanctuary, which
flats, are lateral roots that grow upward
hosts one of the largest rhino
out of the mud and water to function as
populations in the country.
the site of oxygen intake for the submerged
primary root system. • The proposal for de-notification of
any Sanctuary/National Park
Buttress roots, also known as the plank
requires the recommendation of the
roots/stilt roots, are large, wide roots on all
National Board for Wildlife, as per

RAUSIAS-0673 16
https://upscpdf.com/ https://upscpdf.com/
https://t.me/visioniastestseries2024
https://upscmaterial.online/

https://upscpdf.com/ https://upscpdf.com/
the provisions of the Wildlife A fumarole, rich in sulphur gases, is
(Protection) Act, 1972 and called a solfatara.
thereafter, approval from the A fumarole, rich in carbon dioxide, is
Supreme Court also. called a mofette.
• Section 38W of the Wildlife (Protection) Sometimes these features are called the
Act, 1972 "dying volcanoes", because they seem to
Alteration and de-notification of the Tiger represent the last stage of volcanic activity,
Reserves. as the magma, at depth, cools and
o No alteration in the boundaries of a hardens.
Tiger Reserve shall be made, except on
the recommendation of the National 36. Answer: (b)
Tiger Conservation Authority and the Explanation:
approval of the National Board for
Aeolian features include
Wildlife.
all landforms created by the wind
o No State Government shall de-notify a action.
Tiger Reserve, except in the public
• Zeugens are elongated, flat-topped or
interest, with the approval of the
mesa-like ridges that are often found in
National Tiger Conservation Authority
the arid or semi-arid regions. They are
and the National Board for Wildlife.
formed through differential erosion by
wind, where the softer rock is eroded
35. Answer: (c) more quickly than the harder rock,
Explanation: resulting in the characteristic flat-
Fumarole is a vent in the Earth’s topped shape.
surface from which steam and volcanic • Yardangs are streamlined, elongated
gases are emitted. The major source of ridges or hills formed by the wind
the water vapour emitted by the fumaroles erosion of the soft rocks and
is groundwater heated by the bodies of sedimentary deposits. Wind-blown sand
magma lying relatively close to the surface. and particles gradually wear away the
Carbon dioxide, sulphur dioxide and softer rocks, creating these distinctive
hydrogen sulphide are usually emitted features.
directly from the magma. • Dreikanters are specific types of
Fumaroles are often present on active ventifacts, which are the rocks shaped
volcanoes during the periods of relative by wind abrasion. Dreikanters are the
quiet between eruptions. Hydrogen rocks with three distinct faces or sides,
sulphide (H2S), one of the typical gases formed as the wind sculpts them over
issuing from fumaroles, readily oxidizes to time. The process of abrasion by wind-
sulphuric acid and native sulphur. This blown sand can polish and shape the
accounts for the intense chemical rocks, giving them this characteristic
activity and brightly coloured rocks in triangular appearance.
many thermal areas.
Fumaroles are closely related to hot 37. Answer: (a)
springs and geysers. In the areas where the
Explanation:
water table rises near the surface,
Aquatic organisms can be divided into
fumaroles can become hot springs.
three groups, based on where they occur in
the underwater ecosystems.

RAUSIAS-0673 17
https://upscpdf.com/ https://upscpdf.com/
https://t.me/visioniastestseries2024
https://upscmaterial.online/

https://upscpdf.com/ https://upscpdf.com/
Planktons are the numerous, primarily a result of the geological processes
microscopic inhabitants of the pelagic associated with volcanic formations. Crater
environment. They are the critical lakes usually form through the
components of food chains in all marine accumulation of rain, snow and ice melt,
environments, because they provide and groundwater in volcanic craters.
nutrition for the nektons (e.g., crustaceans, Crater lakes can contain fresh water or be
fish and squid) and benthos (e.g., sea warm and highly acidic from hydrothermal
squirts and sponges). fluids.
Planktons are the diverse collection of
organisms found in water, that are
39. Answer: (a)
unable to propel themselves against a
Explanation:
current. For example, phytoplanktons –
Surface inversion promotes stability in
floating and drifting micro plants.
the lower layers of the atmosphere.
Benthic organisms spend their time
Smoke and dust particles get collected
either under, on or near the surface of beneath the inversion layer and spread
the ocean floor. horizontally to fill the lower strata of
Nektons describe the animals that can the atmosphere. Dense fogs in the
swim against water currents, while mornings are common occurrences,
planktons are the plants or animals that especially during the winter season. This
drift in the current. inversion commonly lasts for few hours,
Nektons (or swimmers) are the living until the Sun comes up and begins to
organisms that are able to swim and move warm the Earth.
independently of currents at various
depths of seas and oceans. For example, 40. Answer: (d)
fish and dolphins. Explanation:
Benthos is the community of organisms • The atmosphere is composed of gases,
that live on, in, or near the seabed, also water vapour and dust particles. The
known as the benthic zone. proportion of gases changes in the
higher layers of the atmosphere in
such a way that oxygen will be
38. Answer: (c)
almost in negligible quantity at the
Explanation:
height of 120 km. Similarly, carbon
Crater lakes are the volcanic lakes dioxide and water vapour are found
found in craters and calderas. Crater only upto 90 km from the surface of
lakes are formed by tectonic activity, the Earth.
specifically volcanic activity.
• Carbon dioxide is meteorologically a
These lakes are typically located in volcanic very important gas, as it is
craters or calderas and are characterized transparent to the incoming solar
by a deep basin with steep sides. radiation, but opaque to the outgoing
The basin is often created by the collapse terrestrial radiation. It absorbs a part
of a volcano after a volcanic eruption, and of terrestrial radiation and reflects back
over time, it may fill with water to form a some part of it towards the Earth’s
lake. The steep sides of the crater lakes are surface.

RAUSIAS-0673 18
https://upscpdf.com/ https://upscpdf.com/
https://t.me/visioniastestseries2024
https://upscmaterial.online/

https://upscpdf.com/ https://upscpdf.com/
41. Answer: (b)
Explanation:

42. Answer: (b) hills of the north-eastern region and the


Explanation: Andaman & Nicobar Islands.
Tropical evergreen forests are found on the They are found in warm and humid areas
western slope of the Western Ghats, the with an annual precipitation of over 200

RAUSIAS-0673 19
https://upscpdf.com/ https://upscpdf.com/
https://t.me/visioniastestseries2024
https://upscmaterial.online/

https://upscpdf.com/ https://upscpdf.com/
cm and mean annual temperature above magnolia, laurel, cinchona and wattle.
22 degrees C. Such forests are also found in the
Tropical evergreen forests are well Satpura and the Maikal ranges.
stratified, with layers closer to the ground
and are covered with shrubs and creepers, 43. Answer: (b)
with short-structured trees, followed by tall Explanation:
variety of trees. In these forests, the trees
The Brahmaputra river receives numerous
reach great heights, upto 60 metres or
tributaries in its 750 km long journey
above.
through the Assam valley.
There is no definite time for the trees to
Its major left bank tributaries are the
shed their leaves, flowering and fruition. As
Burhi Dihing and the Dhansari (South),
such, these forests appear green all the
whereas the important right bank
year round. Species found in these forests
tributaries are the Subansiri, the Kameng,
include rosewood, mahogany, aini, ebony,
the Manas and the Sankosh.
etc.
The Sukapaika river is a distributary of
The moist deciduous forests are more
the Mahanadi river.
pronounced in the regions which record
The Penganga, the Indravati, the Pranhita
rainfall between 100 cm - 200 cm. These
and the Manjra are the principal
forests are found in the north-eastern
tributaries of the Godavari river.
states along the foothills of the Himalayas,
eastern slopes of the Western Ghats and Note: The Sukapaika river, which stopped
Odisha. Teak, sal, shisham, hurra, mahua, flowing 70 years ago in Odisha’s Cuttack
amla, semul, kusum and sandalwood etc., district, is set to be rejuvenated, as the
are the main species of these forests. State Government has started working on
its revival plan, following a recent direction
Tendu, palas, amaltas, bel, khair,
from the National Green Tribunal (NGT).
axlewood, etc., are the common trees of
the dry deciduous forest. The river originated from another river, the
Mahanadi, near Ayatpur village. It flowed
In the mountainous areas, the decrease in
27.50 km before meeting the Mahanadi
temperature with increasing altitude leads
again at Bankala.
to a corresponding change in natural
vegetation. Mountain forests can be In the 1950s, the state’s water resource
classified into two types, the northern engineers had, in their wisdom, closed the
mountain forests and the southern Sukapaika river mouth, enabling the
mountain forests. development of the Taladanda Canal
System, a major canal of the state. This led
The southern mountain forests include the
to the river dying a painful death and the
forests found in three distinct areas of the
process was aggravated by agricultural
Peninsular India, viz., the Western Ghats,
encroachments that had sprung up on the
the Vindhyas and the Nilgiris. As they are
river-banks.
closer to the tropics, and only 1,500 m
above the sea level, vegetation is temperate
in the higher regions, and sub-tropical on 44. Answer: (b)
the lower regions of the Western Ghats, Explanation:
especially in Kerala, Tamil Nadu and The Deccan Trap: From the end of the
Karnataka. Cretaceous, till the beginning of the
The temperate forests are called ‘Sholas’ Eocene, stupendous volcanic outbursts
in the Nilgiris, Anaimalai and Palani overwhelmed a vast area of the Peninsular
hills. Some of the other trees of this India, like the one which is seldom known
forest of economic significance include anywhere else in the world’s geological

RAUSIAS-0673 20
https://upscpdf.com/ https://upscpdf.com/
https://t.me/visioniastestseries2024
https://upscmaterial.online/

https://upscpdf.com/ https://upscpdf.com/
history. A vast area of about ten lakh sq. vaginal, anal or oral sex. Cervavac,
km., was flooded by the outpourings of India's first Quadrivalent Human
extremely mobile basalt lava from Papillomavirus Vaccine (qHPV),
fissures and cracks, covering fully the developed by the Serum Institute of
pre-existing topography. These volcanic India, prevents the entry of four of the
deposits have flat top and steep sides, so most common types of HPV (16, 18, 6
that they appear as gigantic steps from a and 11).
distance, and therefore, called ‘Trap’, the ● Pompe disease: A rare and
name derived from the Swedish word, progressive genetic disorder in which
meaning a 'stair’ or 'step'. a complex sugar, called glycogen,
The process of denudation, over a long builds up in the lysosomes. The
period, has reduced the Deccan Trap to autosomal recessive disorder is caused
almost half of its original size and the by a deficiency or malfunction of a
present Deccan Trap covers about 5 lakh specific digestive enzyme, called acid
sq. km., mainly in parts of Kuchchh, alpha-glucosidase (GAA), responsible
Saurashtra, Maharashtra, the Malwa for breaking down glycogen into
Plateau and northern Karnataka. Parts glucose. This leads to the accumulation
of Andhra, Tamil Nadu, Jharkhand and of glycogen, which can lead to
Uttar Pradesh also have some outliers of progressive muscle weakness and can
the Deccan Trap. affect various organs, including heart,
The thickness of the Deccan Traps is respiratory system and liver.
not uniform everywhere. ● Kala-azar: Kala-azar is a slow
It is as much as 3,000 metres along the progressing ‘Neglected Tropical
coast of Mumbai, which is reduced to Disease’, caused by a protozoan
600-800 metres towards the southern parasite of genus Leishmania. The
limit, 800 metres in Kuchchh and only zoonotic infection is transmitted by the
150 metres at Amarkantak, near its sand fly found in moist mud and sand
eastern limit. close to livestock. The parasite
The Deccan Traps have been divided primarily infects the reticuloendothelial
into the following groups: Basalt is the system and may be found in
main rock, but dolerite, rhyolite, gabro and abundance in bone marrow, spleen and
many other rocks are also found. These liver. It is the second-largest parasitic
rocks are a great source of quartz, agate, killer in the world, after Malaria.
calcite, building stones and road building
material. The weathering of these rocks, for 46. Answer: (a)
a long time, has given birth to the black Explanation:
cotton soil, known as 'regur'. ● The Gaganyaan mission envisages
the demonstration of the human
45. Answer: (d) spaceflight capability by launching a
Explanation: crew of 3 members to a Low Earth
● Human papillomavirus (HPV): HPV is Orbit of 400 km for a 3 days’ mission
a small, double-stranded DNA virus, and bring them back safely to the
capable of infecting women and men, Earth, by landing in the Indian sea
both. It can cause cancer in the cervix, waters.
vulva, vagina, penis, anus and throat. ● The crewed flight is planned for 2025
The primary mode of transmission is on the Human Rated Launch Vehicle
through any intimate skin-to-skin Mark 3 as the launch vehicle. It
contact and sexually transmitted via

RAUSIAS-0673 21
https://upscpdf.com/ https://upscpdf.com/
https://t.me/visioniastestseries2024
https://upscmaterial.online/

https://upscpdf.com/ https://upscpdf.com/
comprises solid, liquid and cryogenic ○ ModRNA can help prevent
stages. unwanted immune reactions in the
● The Human Space Flight Centre will body by making the mRNA appear
coordinate the Indian Human more "natural" to the immune
Spaceflight Programme. The agency will system.
be responsible for the implementation ○ Modified bases (ModRNA) can help
of the Gaganyaan mission. stabilize the mRNA, making it last
● The Indian Space Research Organization longer.
(ISRO) has already performed a Crew ○ The process of converting mRNA
Module Atmospheric Re-entry into protein is known as translation.
Experiment and a Pad Abort Test for Modified bases can improve the
the mission. efficiency of translation, leading to
● The project will cost less than Rs. higher protein production.
10,000 crores. ○ ModRNA can be used in vaccine
● If completed on schedule, India will development, protein production
become the world's fourth nation to and developing personalized
conduct independent human medicines.
spaceflight, after the Soviet
Union/Russia, the United States and 48. Answer: (c)
the People's Republic of China. Explanation:
● India envisions a three-phase
47. Answer: (c) programme of nuclear power to ensure
Explanation: energy security.
The Nobel Prize in Medicine for 2023 has ○ First stage: Using Pressurized
been awarded to Katalin Kariko and Drew Heavy Water Reactors (PHWRs)
Weissman for their ground-breaking work utilizing natural Uranium (U-238)
on nucleoside base modification of as fuel and heavy water (deuterium
messenger Ribonucleic Acid (mRNA). Their oxide) as coolant and moderator.
discoveries were critical in the development ○ Second stage: Fast Breeder
of effective mRNA vaccines, including those Reactors (FBRs), using Mixed
used against COVID-19. Oxide (MOX), that combines
● Nucleoside base modification of mRNA plutonium with uranium oxide.
refers to the chemical alteration ○ Third stage: Advanced Heavy Water
(changes to the structure or Reactors (AHWRs) that would utilize
composition) or modification of the Thorium.
individual nucleotide bases [Adenosine ● The FBR can aid in the conversion of
(A), Cytidine (C), Guanine (G) and fertile Thorium 232 to fissile Uranium
Uridine (U)], present in the mRNA 233.
molecule.
● The FBRs can utilize Plutonium 239, a
● Instead of using the standard A, C, G by-product (radioactive waste) from
and U, the scientists can replace them spent Pressurized Heavy Water Reactor
with modified/synthetic bases that (PHWR) fuel, as part of their MOX fuel.
have different chemical properties.
Hence, a nucleoside-modified
49. Answer: (a)
messenger RNA (modRNA) is a
synthetic messenger RNA (mRNA). Explanation:

● Benefits of modRNA include: ● Fullerenes: Fullerene is a family of


carbon allotropes that consists of cage-
RAUSIAS-0673 22
https://upscpdf.com/ https://upscpdf.com/
https://t.me/visioniastestseries2024
https://upscmaterial.online/

https://upscpdf.com/ https://upscpdf.com/
like or tubular structures. The most ● Graphene: Graphene is an allotrope of
well-known fullerene is carbon, consisting of a single layer of
buckminsterfullerene (C60), which carbon atoms arranged in a two-
resembles a soccer ball with 60 carbon dimensional hexagonal lattice.
atoms forming pentagons and Graphene can be rolled-up to form
hexagons. Fullerenes, though not carbon nanotubes (CNTs).
typically abundant, are found ● The applications of Graphene
naturally in soot, volcanic eruptions, include:
interstellar dust clouds and lightning ○ Flexible electronics: Can be used
strikes. in wearable electronics, flexible
● Buckminsterfullerene is the most displays, electronic textiles and
common naturally occurring bendable sensors.
fullerene. Small quantities of it can ○ Energy storage: Enhances
be found in soot. performance and capacity of energy
● The applications of fullerene include: storage systems, like super-
○ Electronics and energy storage: capacitors and high-performance
Can be used in organic batteries.
photovoltaics (solar cells), organic ○ Water purification: Suitable for
light-emitting diodes and organic water filtration and desalination
field-effect transistors. systems.
○ Medicinal chemistry: Can act as ○ Bio-sensors to sense chemical and
anti-oxidants, scavenging free biological agents, explosives,
radicals, targeted drug delivery and radiation and other hazardous
imaging applications, such as substances.
‘magnetic resonance imaging’ (MRI). ○ Aerospace and defence: Develops
○ Catalysts: Can act as catalyst light-weight and high-strength
supports in fuel cells and armour and ballistic protection.
environmental remediation. ○ Medical applications: Bio-sensing,
○ Water purification: Removal of bio-imaging, targeted drug delivery,
pollutants and contaminants due to tissue engineering, phototherapy
their adsorption and catalytic and cancer treatment.
properties.

50. Answer: (b)


Explanation:
● Dark patterns are any practices or deceptive design patterns using user interface or user
experience interactions on any platform that is designed to mislead or trick the users to do
something they originally did not intend or want to do, by subverting or impairing the
consumer autonomy, decision making or choice, amounting to misleading advertisement or
unfair trade practice or violation of the consumer rights.
● The Central Consumer Protection Authority, established under Section 10 of the Consumer
Protection Act, 2019, has notified the ‘Guidelines for Prevention and Regulation of Dark
Patterns, 2023’ to curb dark patterns across various platforms offering goods or services in
India.
● The Guidelines list 13 dark patterns, which include:

RAUSIAS-0673 23
https://upscpdf.com/ https://upscpdf.com/
https://t.me/visioniastestseries2024
https://upscmaterial.online/

https://upscpdf.com/ https://upscpdf.com/
S.N. Dark patterns Definition
1. False Urgency Creates a sense of urgency or scarcity to pressure the
consumers into making a purchase or taking an action.
2. Basket Sneaking Adds additional products or services to the shopping cart
without the user’s consent.
3. Confirm Shaming Involves guilt as a way to make the consumers adhere. It
criticises or attacks the consumers for not conforming to a
particular belief or view-point.
4. Forced Action Forcing the consumers into taking an action they may not want
to take, such as signing up for a service in order to access
content.
5. Subscription Trap Makes it easy for the consumers to sign-up for a service, but
difficult for them to cancel it, often by hiding cancellation option
or requiring multiple steps.
6. Interface Interference Makes it difficult for the consumers to take certain actions, such
as cancelling a subscription or deleting an account.
7. Bait and Switch Practice of advertising a particular outcome, based on the user’s
action, but deceptively serving an alternate outcome.
8. Drip Pricing Involves hiding additional costs from the consumers, until they
are already committed to making a purchase.
9. Disguised Advertisements that are designed to look like other types of
Advertisement content, such as news articles or user-generated content.
10. Nagging Refers to persistent, repetitive and annoyingly constant criticism,
complaints, requests for action.

11. Trick Question Deliberate use of confusing or vague language, like confusing
wording, double negatives, or other similar tricks, to misguide or
misdirect a user.

12. SaaS Billing Process of generating and collecting payments from the
consumers on a recurring basis in a software as a service (SaaS)
business model by exploiting positive acquisition loops in
recurring subscriptions to get money from the users as
surreptitiously as possible.
13. Rogue Malwares Using ransomware or scareware to mislead or trick the users into
believing that there is a virus on their computer and aims to
convince them to pay for a fake malware removal tool that
actually installed malware on their computer.

51. Answer: (a) stem cell therapy or treatment utilizes


Explanation: the stem cells to promote the repair,
● The stem cells are the undifferentiated regeneration, or replacement of the
cells with the potential to develop into damaged or diseased tissues within the
specialized cell types in the body. The body.

RAUSIAS-0673 24
https://upscpdf.com/ https://upscpdf.com/
https://t.me/visioniastestseries2024
https://upscmaterial.online/

https://upscpdf.com/ https://upscpdf.com/
● Types of Stem Cells: SAR-Arcs are diffuse, persistent and
○ Totipotent Stem Cells: Most monochromatic (red in colour), peculiar to
versatile stem cells with the the mid-latitude regions of the Earth. This
potential to differentiate into all the phenomenon is due to the excitation of
different cell types of an organism atomic oxygen by hot electrons.
and the extraembryonic cells A ‘dark sky preserve’ (DSP) is an area,
(including placenta and umbilical usually surrounding a park or an
cord). Only found in the fertilized observatory, that restricts artificial light
egg (zygote). pollution.
○ Pluripotent Stem Cells: Derived
from the inner cell mass of a 53. Answer: (d)
blastocyst, which is an early-stage Explanation:
embryo. They can differentiate
● Quantum dots are the tiny particles or
into all three primary germ layers
nanocrystals of semiconducting
(endoderm, mesoderm and
material made from a semiconductor,
ectoderm), which give rise to all
such as Silicon with a diameter in the
the different cell types in the
range of 2 nm – 10 nm (10 atoms – 50
body (except the extraembryonic
atoms).
cells). For example, the embryonic
● They behave like artificial atoms, as
stem cells and induced pluripotent
they can have a fixed number of
stem cells (reprogrammed from the
electrons in a confined space, leading to
adult cells).
unique properties that are size-
○ Multipotent Stem Cells: More
dependent.
specialized than the pluripotent
Important applications of Quantum Dots
stem cells and can differentiate into
(QDs):
a limited range of cell types within a
specific tissue or organ. For ● Bio-imaging: QDs are 20 times brighter
example, the hematopoietic stem and 100 times more stable than the
cells (give rise to various blood cell traditional fluorescent dyes, are
types), the mesenchymal stem cells photostable and can be used in bio-
(can differentiate into bone, imaging.
cartilage, fat cell etc.) ● Bio-sensors: QD sensors can detect the
○ Unipotent Stem Cells: The most presence of pathogens in food or water,
specialised type of stem cells, can or monitor the levels of pollutants in
only renew themselves and the environment.
differentiate into one specific cell ● Targeted Cancer Treatment: QDs
type. For example, the stem cells in exhibit specific optoelectronic
the skin that can only differentiate properties, which can be utilised for
into another skin cell. targeted cancer treatment. Because of
their nano size, they possess large
surface area, ensuring higher drug
52. Answer: (a)
loading capacity and can tag
Explanation:
nanocarriers in the biological systems.
‘Stable Auroral Red Arcs’ (SAR-Arcs) are
● Optical Applications: QDs can be used
aurora like phenomena which result in
to make smaller and more efficient
the display of vivid shades of red,
image sensors, like the CMOS
instead of more common green and blue
(Complementary Metal Oxide
shades in the Polar auroras.
Semiconductor) sensors. The QLEDs

RAUSIAS-0673 25
https://upscpdf.com/ https://upscpdf.com/
https://t.me/visioniastestseries2024
https://upscmaterial.online/

https://upscpdf.com/ https://upscpdf.com/
(Quantum-dot LEDs) are capable of Anusandhan National Research
emitting all colours depending on their Foundation (ANRF), which will come into
size. Thus, they provide high-definition, being after notification from the Central
brighter and more colourful displays. Government.
● Flexible Electronics: Quantum-dot Objectives of the Anusandhan National
logic circuits layout building blocks for Research Foundation (ANRF):
innovative devices, including printable • The ANRF will serve as the apex body to
electronics and flexible display screens. provide a high-level strategic direction
for research, innovation and
54. Answer: (c) entrepreneurship in India in the fields
Explanation: of natural sciences, mathematical
sciences, engineering and technology,
• Picoflare jets are relatively small jets
environmental and earth sciences,
of charged particles expelled
health and agriculture and scientific
intermittently from the Sun’s corona
and technological interfaces of
at supersonic speeds for 20 to 100
humanities and social sciences.
seconds.
• Preparing a short-, medium- and long-
• These charged jets are a source of the
term research and development.
solar wind.
• Seeding, growing and facilitating
• These jets from the Sun are named so
research at academic and research
because they carry approximately one-
institutions, particularly at universities
trillionth energy of the largest flares the
and colleges where research capacity is
Sun is capable of producing. “Pico” is
at a nascent stage, through
an order of magnitude, that denotes 10-
12 or one trillionth of a unit.
programmes, such as R&D projects,
fellowships, academic chairs and
• It has significant effects on the large-
creation of centres of excellence.
scale solar system, as well as on the
• Funding competitive peer-reviewed
Earth’s magnetic field and poses risks
grant proposals to the eligible persons.
to the satellites.
• Assisting in setting-up research
infrastructure and environment that is
55. Answer: (a)
conducive for scientific pursuit, with
Explanation:
specific focus on the matters of national
The Anusandhan National Research priorities, emerging frontiers and
Foundation Act, 2023: The Parliament strategic research.
has enacted the Anusandhan National
• Increasing India's role and participation
Research Foundation Act (2023) to provide
in the key areas of national and global
a high-level strategic direction for research,
importance.
innovation and entrepreneurship in the
• Supporting translation of research
fields of natural sciences, mathematical
undertaken into capital intensive
sciences, engineering and technology,
technologies.
environmental and earth sciences, health
and agriculture and scientific and • Evolving nationally coordinated
technological interfaces of humanities and programmes to identify scientific and
social sciences to promote, monitor and practical solutions for societal,
provide support for research in India. developmental, financial and techno-
economic challenges.
Salient Features of the Anusandhan
National Research Foundation Act, • Coordinating across the Central
2023: The Act establishes a statutory Government, the State Governments,

RAUSIAS-0673 26
https://upscpdf.com/ https://upscpdf.com/
https://t.me/visioniastestseries2024
https://upscmaterial.online/

https://upscpdf.com/ https://upscpdf.com/
public authorities, industries and The Principal Scientific Advisor to
research institutions to document and the Government of India will be the
analyze expenditure on scientific ex-officio Member Secretary of the
research and their outcomes during Governing Board.
each financial year, and report the • Executive Council:
same to the Central Government. o The President of the Governing
• Evolving participation in international Board shall constitute an Executive
collaborative projects and fostering Council to implement the provisions
exchange of scientific information. of this Act.
• Encouraging collaboration with the o The Principal Scientific Advisor to
scientists from within and outside the Government of India will be the
India, including the scientists of Indian ex-officio Chairperson of the
origin, with a view to enrich the Indian Executive Council. The Chief
scientific ecosystem. Executive Officer (CEO) of the ANRF
• Encouraging Public Sector Enterprises will be the Member Secretary of this
and private sector entities to invest in body.
the activities of the ANRF. o Functions of the Executive Council:
• The ANRF shall also, to the extent The Executive Council shall implement the
practicable, either by itself or through a objectives of the ANRF, based on policy
suitable agency, undertake an annual direction and guidance provided by the
survey of the outcomes of the scientific Governing Board. The functions of the
research in India, to create a central Executive Council are:
repository for collection, interpretation • Consider the applications for grant of
and analysis of information. The aim of financial assistance in accordance with
this repository would include providing the eligibility criteria, as determined
information for policy formulation and necessary for such grants.
advising the Central Government and • Facilitate and provide any assistance,
the State Governments, and the private
as may be required to ensure filing of
sector. This survey will exclude any
the applications for IPR (Intellectual
strategic areas of research as Property Right), pursuant to the
determined by the Governing Board.
research undertaken through financial
Institutions created under the assistance.
Anusandhan National Research
• To determine through regulations:
Foundation Act (2023):
o The requirements for registration,
• Governing Board:
form and manner for making of the
o The Governing Board shall provide applications for financial assistance.
high-level strategic direction,
o Reports and certifications to be
perform and monitor the
furnished, pursuant to availing the
implementation of the objectives of financial assistance.
the ANRF.
o The requirements of the extension
o The Prime Minister of India shall be of the financial assistance.
the ex-officio President of the
o Grounds for revocation of the
Governing Board. The Union
financial assistance.
Minister of Science and Technology
and the Union Minister of • Chief Executive Officer: The President
Education will be the ex-officio Vice- of the Governing Board may appoint a
Presidents of the Governing Board. Chief Executive Officer, not below the
rank of an Additional Secretary to the

RAUSIAS-0673 27
https://upscpdf.com/ https://upscpdf.com/
https://t.me/visioniastestseries2024
https://upscmaterial.online/

https://upscpdf.com/ https://upscpdf.com/
Government of India for efficient admission procedure and fee fixation do
administration of the ANRF. not violate the right of the minorities under
Article 30(1) or the right of the minorities
56. Answer: (c) and the non-minorities under Article 19(1)
(g). They are reasonable restrictions in the
Explanation:
interest of the minority institutions
Article 15(5) prohibits state to special
permissible under Article 30(1) and in the
provision, by law, for the advancement of
interest of the general public under Article
any socially and educationally backward
19(6) of the Constitution.”
classes of citizens or for the Scheduled
The Islamic Academy Case: Justice
Castes or the Scheduled Tribes in so far as
Sinha explained profiteering with the help
such special provisions relate to their
of the Black’s Law Dictionary – ‘Taking
admission to minority educational
advantage of unusual or exceptional
institutions referred to in clause (1) of
circumstances to make excessive profits’.
article 30.
Action Committee Unaided Private
Moreover, P.A. Inamdar is an authority on
Schools of Delhi v/s the Director of
the proposition of law that neither can the
Education: On a perusal of the T.M.A Pai
policy of reservation be enforced by the
Foundation and P.A. Inamdar, it can be
State, nor can any quota or percentage of
inferred that private unaided institutions
admission be carved-out to be appropriated
are permitted to have a profit, but not
by the State in a minority educational
permitted to profiteer.
institution.
The State cannot regulate or control
admissions in the minority educational 57. Answer: (a)
institutions, so as to compel them to give Explanation:
up a share of the available seats to the A Constitutional Tort is a violation of
candidates chosen by the State. This would one’s constitutional rights by a
amount to the nationalisation of the seats, government employee. The alleged
which has been specifically disapproved in constitutional violation creates a cause of
T.M.A. Pai. action that is distinct from any otherwise
Such imposition of quota of the State available state tort remedy.
seats, or enforcing the reservation policy of In India, the history of Constitutional Tort
the State on the available seats in the can be traced back to the Acts enacted by
minority educational institutions, are the the British East India Company.
acts constituting a serious encroachment It initially found a place in Section 65 of
on the right enshrined in Article 30(1). the Government of India Act of 1858. This
Such appropriation of seats can also not be was further inherited in Section 176 of the
held to be a regulatory measure or a Government of India Act of 1935. Thus,
reasonable restriction within the meaning Section 176 formed the basis and Article
of Article 30(1) of the Constitution. 300 of the Constitution of India emerged
The T.M.A. Pai Foundation: Every from this.
institution is free to devise its own fee Article 300 of the Constitution provides for
structure, subject to the limitation that the suits and proceedings to be instituted
there can be no profiteering and no against the State in the name
capitation fee can be charged directly or of the Union of India. The tortuous liability
indirectly. of the State, thus, arises from the vicarious
P.A. Inamdar: Legal provisions made by liability of its servants in performance of
the State Legislatures or the scheme non-sovereign functions.
evolved by the Court for monitoring the
RAUSIAS-0673 28
https://upscpdf.com/ https://upscpdf.com/
https://t.me/visioniastestseries2024
https://upscmaterial.online/

https://upscpdf.com/ https://upscpdf.com/
58. Answer: (b) (Twelfth Amendment) Act, 1962, under
Explanation: which the Territory of Goa, Daman and Diu
The Constitution (Eighteenth Amendment) was included in the First Schedule to the
Act, 1966, added explanations to Article 3, Constitution of India and was declared as a
which state as follows: Union Territory.
Article 3: Formation of new States and The statement of Objects and Reasons of
alteration of areas, boundaries or names of the Constitution (Twelfth Amendment) Act,
the existing States — The Parliament may 1962, states as follows:
by law: On the acquisition of the territories of Goa,
(a) form a new State by separation of Daman and Diu with effect from the 20th
territory from any State or by uniting December, 1961, these territories have, by
two or more States or parts of States or virtue of sub-clause (c) of clause (3) of Article
by uniting any territory to a part of any 1 of the Constitution, been comprised within
State; the territory of India from that date and they
are being administered as a Union Territory
(b) increase the area of any State;
by the President through an Administrator
(c) diminish the area of any State;
in accordance with Article 239 of the
(d) alter the boundaries of any State; Constitution. It is, however, considered
(e) alter the name of any State: desirable that Goa, Daman and Diu should
Provided that no Bill for the purpose shall be specifically included as a Union Territory
be introduced in either House of the in the First Schedule to the Constitution. It is
Parliament, except on the recommendation also considered that clause (1) of Article 240
of the President and unless, where the should be suitably amended to confer power
proposal contained in the Bill affects the on the President to make regulations for the
area, boundaries or name of any of the peace, progress and good government of
States , the Bill has been referred by the Goa, Daman and Diu, as has been done in
President to the Legislature of that State the case of Dadra and Nagar Haveli.
for expressing its views thereon within The issue of conversion of Jammu and
such period as may be specified in the Kashmir into two Union Territories was
reference or within such further period as filed in the Supreme Court, whereby the
the President may allow and the period so Court held that the Parliament has the
specified or allowed has expired. power to convert State into Union Territory,
Explanation I: In this Article in Clauses (a) alter boundaries under Article 3 and the
to (e), "State" includes a Union Territory, explanations provided thereunder. The
but in the proviso, "State" does not include Court said that such law cannot be
a Union Territory. considered as an amendment to the
Explanation II: The power conferred on Constitution. On a conjoint reading of
the Parliament by Clause (a) includes the Articles 3, 4 and 239A, the Court found
power to form a new State or Union that “the Parliament, by making a law, can
Territory by uniting a part of any State or convert an existing State into one or more
Union Territory to any other State or Union Union Territories.” Explaining the
Territory. constitutional scheme, the Bench said,
The Supreme Court has held that a new “Article 3 provides that the Parliament may
territory, as a Union Territory, can only be by law form new States and alter the areas,
admitted and established through an boundaries or names of the existing States.
amendment of the Constitution under Explanation I provides that in Clauses (a)
Article 368. Accordingly, the Indian to (e) of Article 3, a “State” includes “Union
Parliament passed the Constitution Territory”. Thus, explanation I makes it

RAUSIAS-0673 29
https://upscpdf.com/ https://upscpdf.com/
https://t.me/visioniastestseries2024
https://upscmaterial.online/

https://upscpdf.com/ https://upscpdf.com/
amply clear that the power of the or to an hour or part of the same day.
Parliament, under Clause (a) of Article 3, to Provided that the Speaker, if he thinks
make a law to form a new State or to alter fit, may call a sitting of the House
a boundary of a State, includes a power to before the date or time to which it has
make a law to form a new Union Territory. been adjourned or at any time after the
Explanation II clarifies that the power House has been adjourned sine die.
conferred by Clause (a) on the Parliament
to enact the law to form a new State 60. Answer: (c)
includes a power to form a Union Territory Explanation:
by uniting parts of any State or Union
The Election Symbols (Reservation and
Territory to any other State or Union
Allotment) Order, 1968, has empowered
Territory.
the Election Commission (EC) to allot
symbols at elections in the Parliamentary
59. Answer: (b) and the Assembly Constituencies,
Explanation: recognition of the political parties and
▪ Adjournment of a House does not suspend or withdraw recognition of
terminate the session of the House. It recognised or unrecognised political party
merely postpones the proceedings of for its failure to observe the Model Code of
the House to a future date. Conduct or follow lawful instructions of the
▪ Adjournment only terminates the EC.
sitting of the House and not the According to the Election Symbols Order,
session. 1968, "Political Party" - means an
▪ Prorogation brings an end to a session association or body of individual citizens of
of the House done by an order of the India registered with the Commission as a
President under Article 85(2)(a). political party, under Section 29A of the
Representation of the People Act, 1951.
▪ Prorogation of the House may take
place any time, even while the House is Rule 6 provides for the classification of the
sitting. However, usually, Prorogation political parties —
follows the adjournment of the sitting of ▪ For the purposes of this Order and for
the House sine die. such other purposes as the
▪ Prorogation terminates both the sitting Commission may specify as and when
and the session of the House. necessity therefore arises, the political
parties are either recognised political
▪ Adjournment sine die refers to the
parties or unrecognised political
adjournment of the House of the
parties.
Parliament for an indefinite period,
without assigning any specific day or ▪ A recognised political party shall either
date for its reconvening. be a National party or a State party.
▪ The power of Adjournment sine die lies Rule 15: The power of the Commission
with the Presiding Officer of the House in relation to splinter groups or rival
– The Chairman/the Speaker. sections of a recognised political party:
Rule 15 of the Rules of Procedure and When the Commission is satisfied on the
Conduct of Business in Lok Sabha information in its possession that there are
mentions about the adjournment of the rival sections or groups of a recognised
House and procedure for reconvening. It political party, each of whom claims to be
states that the Speaker shall determine the that party the Commission may, after
time when a sitting of the House shall be taking into account all the available facts
adjourned sine die or to a particular day, and circumstances of the case and hearing

RAUSIAS-0673 30
https://upscpdf.com/ https://upscpdf.com/
https://t.me/visioniastestseries2024
https://upscmaterial.online/

https://upscpdf.com/ https://upscpdf.com/
such representatives of the sections or ground of proved misbehaviour or
groups and other persons as desire to be incapacity.
heard, decide that one such rival section or Article 312: All-India Services: (1)
group or none of such rival sections or Notwithstanding anything in Chapter VI of
groups is that recognised political party Part VI or Part XI, if the Council of States
and the decision of the Commission shall has declared by resolution supported by
be binding on all such rival sections or not less than two-thirds of the members
groups. present and voting that it is necessary or
Rule 16: The power of the Commission expedient in the national interest so to do,
in case of amalgamation of two or more the Parliament may by law provide for the
political parties: creation of one or more All-India Services
1. When two or more political parties, one (including an All-India Judicial Service)
or some or all of whom is a recognised common to the Union and the States, and,
political party or are recognised subject to the other provisions of this
political parties join together to form a Chapter, regulate the recruitment, and the
new political party, the Commission conditions of service of persons appointed,
may, after taking into account all the to any such service.
facts and circumstances of the case,
hearing such representatives of the 62. Answer: (b)
newly formed party and other persons Explanation:
as desired to be heard and having
The National Investigation Agency (NIA)
regard to the provisions of this Order,
is the primary counter-terrorist task force
decide— (a) whether such newly formed
of India. The Agency is empowered to deal
party should be a National party or a
with the investigation of terror-related
State party; and (b) the symbol to be
crimes across the States, without special
allotted to it.
permission from the States, under written
2. The decision of the Commission, under Proclamation from the Ministry of Home
sub-paragraph (1), shall be binding on Affairs.
the newly formed political party and all
Governing Body: The Ministry of Home
the component units thereof.
Affairs, the Government of India.
The Serious Fraud Investigation Office
61. Answer: (d) (SFIO) is a statutory corporate fraud
Explanation: investigating agency in India. Initially, it
Article 61: Procedure for the was set-up by a resolution adopted by the
impeachment of the President: Such Government of India in 2003, and carried
resolution has to be passed by a majority out investigations within the existing legal
of not less than two-thirds of the total framework under Sections 235 to 247 of
membership of the House. the erstwhile Companies Act, 1956. Later,
Article 124 (4): A Judge of the Supreme Section 211 of the Companies Act, 2013,
Court shall not be removed from his office accorded the statutory status to the
except by an order of the President passed Serious Fraud Investigation Office (SFIO).
after an address by each House of the It is under the jurisdiction of the
Parliament, supported by a majority of the Ministry of Corporate Affairs, the
total membership of that House and by a Government of India, and primarily
majority of not less than two-thirds of the supervised by the officers from the Indian
members of that House present and voting Administrative Service; the Indian Police
has been presented to the President in the Service; the Indian Corporate Law Service;
same session for such removal on the the Indian Revenue Service; and other

RAUSIAS-0673 31
https://upscpdf.com/ https://upscpdf.com/
https://t.me/visioniastestseries2024
https://upscmaterial.online/

https://upscpdf.com/ https://upscpdf.com/
Central Services. The organization has any offence shall be compelled to be a
experts from various financial sector witness against himself.
domains. The SFIO is mandated to conduct • Under Article 22, the Constitution also
multi-disciplinary investigations of major provides for protection against arrest
corporate frauds. and detention in certain cases. Every
The Central Bureau of Investigation person, who is arrested, shall be
(CBI) is the premier investigating agency of informed of the grounds of arrest and
India. It operates under the jurisdiction of shall have a right to consult and to be
the Ministry of Personnel, Public defended by a legal practitioner of his
Grievances and Pensions. Originally set-up choice. Every person, who is arrested
in 1965 to investigate bribery and and detained in custody, shall be
governmental corruption, it received produced before the nearest magistrate
expanded jurisdiction to investigate within a period of 24 hours of such
breaches of the central laws enforceable by arrest.
the Government of India, multi-state • However, these provisions under Article
organized crime, multi-agency or 22 shall not be applicable to –
international cases. The CBI has attracted o any person, who, for the time being,
numerous controversies and criticisms due
is an enemy alien;
to various reports of irregular practices,
o any person who is arrested or
excessive political influence and a poor
detained under any law providing
conviction rate. The CBI is exempted from
for preventive detention.
the provisions of the Right to Information
Act (2005). The CBI is India's officially
designated single point of contact for 64. Answer: (d)
liaison with the Interpol (The International Explanation:
Criminal Police Organization). The Deputy Prime Minister is appointed
under Article 75 of the Indian Constitution,
63. Answer: (b) whereas the Deputy Chief Minister is
appointed under Article 164 of the
Explanation:
Constitution.
• The Indian Evidence Act, 1872: The
But the posts of the Deputy Prime
confession by the accused while in
Minister and the Deputy Chief Minister
the custody of the police not to be
are not mentioned in the Constitution.
proved against him: The confession
made by any person while he is in • The Supreme Court, in the ‘K.M.
the custody of a police officer shall Sharma v/s Devi Lal and Ors (1990)’
not be proved as against such case opined that the terminology
person, unless it be made in the “Deputy” is only descriptive and does
immediate presence of a magistrate. not confer on the Deputy Prime
Minister any power of the Prime
• As per Article 20, a person shall be
Minister. He is like any other Cabinet
convicted only for the violation of a law
Minister.
which is in force at the time of the
commission of the crime and penalized • Similarly, in a recent PIL case, the
accordingly. It means that he shall not Supreme Court opined that even a
be penalized more for any crime Deputy Chief Minister was a Minister
committed by the person. Further, no first, and that the post of a Deputy
person shall be prosecuted and Chief Minister was “only a label”.
punished for the same offence more • In the ‘Official Table of Precedence’,
than once, and no person accused of the Deputy Prime Minister is ranked

RAUSIAS-0673 32
https://upscpdf.com/ https://upscpdf.com/
https://t.me/visioniastestseries2024
https://upscmaterial.online/

https://upscpdf.com/ https://upscpdf.com/
above the Union Cabinet Ministers first Auditor-General, Sir Edward
and the Deputy Chief Minister is Drummond, in 1860.
ranked above the State Cabinet Under the Mont–Ford Reforms of 1919,
Ministers. the Auditor-General became
independent of the Government.
65. Answer: (d) The Government of India Act, 1935,
Explanation: strengthened the position of the
The 73rd Amendment Act (1992) does Auditor-General.
not directly provide for the reservation Article 148 provides for the
of the seats in the favour of the appointment of the Comptroller and
‘backward classes’. Auditor-General of India (CAG) only at
Article 276 of the Constitution: Taxes the Centre and not at the States.
on professions, trades, callings and The CAG is appointed by the President by
employments: warrant under his hand and seal, and
(1) Notwithstanding anything in Article shall only be removed from the office in like
246, no law of the Legislature of a manner and on the like grounds as a
State, relating to taxes for the benefit of Judge of the Supreme Court.
the State or of a municipality, district As per Article 148(4), the Comptroller and
board, local board or other local Auditor-General shall not be eligible for
authority therein in respect of further office, either under the Government
professions, trades, callings or of India, or under the Government of any
employments, shall be invalid on the State, after he has ceased to hold his office.
ground that it relates to a tax on
income. 67. Answer: (a)
(2) The total amount payable in respect of Explanation:
any one person to the State or to any Article 174 (1) of the Constitution: The
one municipality, district board, local Governor shall from time to time
board or other local authority in the summon the House or each House of the
State by way of taxes on professions, Legislature of the State to meet at such
trades, callings and employments, shall time and place as he thinks fit, but six
not exceed [two thousand and five months shall not intervene between its
hundred rupees] per annum. (beyond last sitting in one session and the date
the 73rd Amendment Act). appointed for its first sitting in the next
According to Article 243E of the session.
Constitution: Article 175 (1) of the Constitution: The
(4) A Panchayat, constituted upon the Governor may address the Legislative
dissolution of a Panchayat before the Assembly or, in the case of a State having a
expiration of its duration, shall Legislative Council, either House of the
continue only for the remainder of the Legislature of the State, or both Houses
period for which the dissolved assembled together, and may for that
Panchayat would have continued under purpose require the attendance of the
Clause (1) had it not been so dissolved. members.
Each House (if a State has two Houses) is
66. Answer: (a) obligated to provide in its rules for the
Explanation: procedure for the allotment of time for
The Government of India Act, 1858, discussing the Governor’s address.
paved way for the appointment of the

RAUSIAS-0673 33
https://upscpdf.com/ https://upscpdf.com/
https://t.me/visioniastestseries2024
https://upscmaterial.online/

https://upscpdf.com/ https://upscpdf.com/
The Constitution makes no provision to A Financial Bill is treated like any other
meet the contingency arising out of the Ordinary Bill by both the Houses. The
Governor’s unavoidable absence due to Rajya Sabha has full power to effect
illness on the occasion of the opening of amendments in it and a deadlock between
the Legislature. the two Houses will have to be resolved
The Speaker, in consultation with the through the procedure of a ‘joint session’.
Leader of the House, shall allot time for the MONEY BILL - ARTICLE 110(1)(f) - the
discussion of the matters, referred to in the receipt of money on account of the
Governor’s address at the commencement Consolidated Fund of India or the Public
of session of the Vidhan Sabha under Account of India or the custody or issue of
Article 176 (1) of the Constitution. such money or the audit of the accounts of
the Union or of a State.
68. Answer: (b)
Explanation: 70. Answer: (b)
Article 251 of the Constitution: Nothing Explanation:
in Articles 249 and 250 shall restrict the Article 105 of the Constitution has four
power of the Legislature of a State to make clauses –
any law which under the Constitution it Clause (1) declares that there shall be
has power to make, but if any provision of freedom of speech in the Parliament.
a law made by the Legislature of a State is This freedom is subject to the
repugnant to any provision of a law made Constitution, and to the rules and
by the Parliament which the Parliament standing orders regulating the
has under either of the said Articles power procedure in the Parliament. Therefore,
to make, the law made by the Parliament, the freedom of speech in the Parliament
whether passed before or after the law would be subject to the provisions that
made by the Legislature of the State, shall regulate its procedure framed under
prevail, and the law made by the Article 118.
Legislature of the State shall to the extent It is also subject to Article 121 which
of the repugnancy, but so long only as the restricts the Parliament from discussing
law made by the Parliament continues to the conduct of any Judge of the Supreme
have effect, be inoperative. Court or of a High Court in the discharge
Article 253 of the Constitution: The of his duties, except upon a motion for
Parliament has power to make any law for presenting an address to the President
the whole or any part of the territory of praying for the removal of the Judge.
India for implementing any treaty, In interpreting these Privileges, therefore,
agreement or convention with any other regard must be given to the general
country or countries or any decision made principle that the Privileges of the
at any international conference, Parliament are granted to the members in
association or other body. order that "they may be able to perform
their duties in the Parliament without let
69. Answer: (c) or hindrance". These apply to the
Explanation: individual members "only insofar as they
Both the Financial Bill and the Money are necessary in order that the House may
Bill originate only in the Lok Sabha. freely perform its functions”.
Neither can be introduced without the The Supreme Court held that the
recommendation of the President of Parliament or the Legislature is not the
India. sole judge of its Privileges, and the
courts have the power to enquire if a
RAUSIAS-0673 34
https://upscpdf.com/ https://upscpdf.com/
https://t.me/visioniastestseries2024
https://upscmaterial.online/

https://upscpdf.com/ https://upscpdf.com/
particular Privilege claimed by the 72. Answer: (c)
Legislature in fact existed or not, by Explanation:
consulting the Privileges of the • The Reserve Bank of India (RBI)
Commons. regulates money supply in the economy
The determination of Privileges, the to achieve certain objectives, like price
court held, and whether they conform to stability, accelerating economic growth
the parameters of the Constitution is a or exchange rate stabilization. The RBI
question that must be answered by the does it by changing the key policy rates,
courts. thereby forcing the banks to alter their
deposit and lending rates.
71. Answer: (d) • The opportunity cost of holding cash by
Explanation: the public alters, depending upon the
• The Central Bank Digital Currency prevailing interest rates in the market.
(CBDC) refers to the legal tender issued The opportunity cost of holding cash
by a Central Bank in a digital form. The increases with increasing interest
CBDCs can be distributed directly by rates, as the holder misses out an
the Central Bank and the payments in opportunity to earn interest on the
the CBDC involve simple transfers from money had it been deposited in the
one account to another, while it bank.
remains a direct claim on the Central
Bank which issued it. This model is 73. Answer: (b)
referred to as the ‘direct model of Explanation:
distribution’.
• Liquidity in the banking system refers
• However, in the indirect model, the to the readily available cash that the
Central Bank will pass the digital banks need to meet short-term
currency token to a commercial bank or business and financial needs. On a
a non-banking financial institution given day, if the banking system is a
(e.g., Fintech), which will then net borrower from the Reserve Bank of
distribute the currency and also handle India (RBI) under the Liquidity
the ‘KYC’ (Know Your Customer) Adjustment Facility (LAF), the system
requirements. The claim for the liquidity can be said to be in deficit,
currency will be on the commercial and if the banking system is a net
bank or the non-bank financial lender to the RBI, the system liquidity
institution, and not on the Central can be said to be in surplus.
Bank. This type of CBDC is also • Recently, the liquidity deficit in the
referred to as ‘synthetic CBDC’. banking system rose to over 13-year
These synthetic CBDCs are backed high.
by matching reserves with the
• The possible reasons for ‘banking
Central Bank.
system liquidity deficit’:
• India adopted a hybrid model
o Increasing demand for currency:
(Intermediate model of CBDC), where
Increasing demand for the currency
the Central Bank distributes the CBDC
by the public reduces the deposits
to a regulated intermediary, such as a
in the banks.
commercial bank or a Fintech, which
o Increasing demand for credit,
handles the transaction and the KYC
which is reflected in the Credit-
and AML (Anti-money laundering)
Deposit Ratio, a metric that gauges
requirements. However, importantly,
the proportion of a bank's lending in
the claim remains on the Central Bank.

RAUSIAS-0673 35
https://upscpdf.com/ https://upscpdf.com/
https://t.me/visioniastestseries2024
https://upscmaterial.online/

https://upscpdf.com/ https://upscpdf.com/
relation to the deposits it 76. Answer: (a)
accumulates. Increasing Credit- Explanation:
Deposit Ratio may result in Salient Features of the Competition
decreasing the banking system (Amendment) Act, 2023:
liquidity.
• The Act prohibits combinations like
o Forex intervention by the RBI to mergers, acquisitions or amalgamations
arrest the depreciating value of the of the enterprises whose transaction
rupee may reduce liquidity with the value is above Rs. 2,000 crores, and
banking. which may cause an adverse effect on
competition. Thus, any combination,
74. Answer: (c) whose transaction value is above Rs.
Explanation: 2,000 crores, needs to be approved
‘Connected lending’ refers to a banking by the Competition Commission of
system in which the bank owner India (CCI), which gives approval on
provides loans to its own company, a case-by-case basis.
affiliated entities, or connected • The approval of combination from the
individuals (such as friends and family) CCI is required to be given within 150
at reduced interest rates. Connected days.
lending can involve moral hazard issues, • The Act also prohibits anti-
leading to compromise in pricing and credit competitive agreements, which may
management. The Reserve Bank of India include directly or indirectly
has decided to tighten the connected determining the sale price; collusive
lending norms to prevent instances where bidding; controlling production or
the persons in power, who influence supply etc. However, such
lending decisions, have close connections agreements are not only confined to
with the borrowers. the enterprises engaging in similar
or identical businesses, but extended
75. Answer: (a) to the enterprises who are not
Explanation: engaged in similar businesses.

• As per Article 293, a State may not, • The Amendment introduced “leniency
without the consent of the Government plus” regime, where the CCI can reduce
of India, raise any loan if there is still penalty imposed by it on any enterprise
outstanding any part of a loan which for an exchange of information on any
has been made to the State by the other cartel engaged in anti-competitive
Government of India. arrangements.

• The Centre can give guarantees to


other government agencies within 77. Answer: (a)
the limits fixed by the Parliament. Explanation:
Hence, the Fiscal Responsibility and • A ‘derivative’ is a financial instrument
Budget Management (FRBM) Act (2003) whose value is based upon the value of
fixed a limit on such guarantees to an underlying asset, like equities,
0.5% of the GDP in a fiscal year. The currency or commodities, or other
Union FRBM Act,2003 does not impose financial assets. A ‘derivative contract’,
any restrictions on guarantees by which has a commodity as its
states. States guarantees are regulated underlying, is known as a ‘commodity
their respective state FRBM derivative’ contract. Most common
legislations. types of derivative instruments are

RAUSIAS-0673 36
https://upscpdf.com/ https://upscpdf.com/
https://t.me/visioniastestseries2024
https://upscmaterial.online/

https://upscpdf.com/ https://upscpdf.com/
‘forwards’, ‘futures’, ‘options’ and market. In a ‘future contract’, the
‘swaps’. buyer has the obligation to buy/sell
• ‘Futures’ and ‘options’ are the two the assets. On the other hand, in an
important derivative instruments that ‘option contract’, the customer has
are traded in the commodity derivative no obligation to buy/sell the assets.

78. Answer: (b)


Explanation:
• Non-Resident External (NRE) Accounts: The Non-Resident Indians (NRIs) can deposit their
foreign income (converted into Indian currency) in the NRE accounts, which are maintained
in Indian rupees.
• Foreign Currency Non-Resident (FCNR) accounts: These accounts allows the NRIs to save
their foreign income in foreign currency.
Non-Resident External (NRE) Foreign Currency Non-
Account Resident (FCNR) Account
Who can Open? Non-Resident Indians (NRIs)/ NRIs/PIOs/OCIs
Persons of Indian Origin (PIOs)/
Overseas Citizens of India
(OCIs)
Deposits Foreign currency Foreign currency
Withdrawals Indian rupee Foreign currency
Exchange rate risk Foreign exchange risk due to No foreign exchange risk
currency fluctuations
Repatriability Deposits and interest earned Deposits and interest earned
are fully repatriable are fully repatriable
Taxation in India Not taxable in India Not taxable in India

79. Answer: (d) 80. Answer: (a)


Explanation: Explanation:
• FCI sells surplus stocks of wheat and • Buyback is the repurchase by a
rice under Open Market Sale Scheme company of its shares from the existing
(Domestic) at pre-determined prices shareholders that reduces the number
through e-auction in the open market of its shares in the open market. This is
from time to time to enhance the one of the routes taken by the
supply of food grains and thereby government to meet its disinvestment
moderate the open market prices. targets.
• Usually state governments are allowed
• The company buy back the shares held
to participate in the scheme along with
by the government from its excess
the private traders. But, states are not
reserves and cancels them, thereby
mandated to participate in the scheme.
reducing the total number of shares
Recently, Union Food Ministry
held and total share capital. However,
discontinued the sale of rice and wheat
buyback would result in increasing
under the Open Market Sale Scheme
share of remaining shareholders.
(OMSS) to all state governments to
control the rising food prices.

RAUSIAS-0673 37
https://upscpdf.com/ https://upscpdf.com/
https://t.me/visioniastestseries2024
https://upscmaterial.online/

https://upscpdf.com/ https://upscpdf.com/
81. Answer: (b) sector should be given to the service
Explanation: providers from all other WTO members.
• The General Agreement on Trade in ▪ Transparency: The GATS says that the
Services (GATS) is a treaty administered governments must publish all relevant
by the World Trade Organization (WTO) laws and regulations, and set-up
that aims to create a framework for the enquiry points within their
liberalization of international trade in bureaucracies. The foreign companies
services. and the governments can then use
• All members of the WTO are the parties these inquiry points to obtain
information about regulations in any
to the GATS. The fundamental
service sector.
principles of the GATS apply, in
principle, to all service sectors, o Specific commitments: Apply only
except: to the sectors inscribed in a
o services supplied in the exercise of
member's schedule of commitments
governmental authority on a non- with respect to ‘market access’ and
‘national treatment’. Under the
commercial basis, such as social
security schemes, public health, Positive List Approach, the countries
specify the sectors or services in
education.
which they are willing to make
o services related to air transport.
commitments to liberalize trade.
• The obligations contained in the This means that only those services,
GATS may be categorized into two explicitly listed in a country's
broad groups: schedule of commitments, are
o General obligations: Apply to all subject to liberalization obligations
members and services sectors. and disciplines under the GATS.
▪ Most Favoured Nation: Under the Any services not included in the
GATS, if a country allows foreign schedule are not subject to the
competition in a sector (for example, GATS rules.
education), equal opportunities in that

82. Answer: (c)


Explanation:
Trends in Cropping Pattern:

RAUSIAS-0673 38
https://upscpdf.com/ https://upscpdf.com/
https://t.me/visioniastestseries2024
https://upscmaterial.online/

https://upscpdf.com/ https://upscpdf.com/

Year 2014-15 2015-16 2016-17 2017-18 2018-19 2019-20 2020-21 2021-22 2022-23 2023-24
Wheat 865.27 922.88 985.10 998.70 1035.96 1078.61 1095.86 1077.42 1105.54 1120.19
Production
Important Observations:
● Rice and wheat account for the largest area under cultivation.
● Stagnation in the percentage of area under wheat in the last decade.
● Increase in the production of wheat, despite stagnation in the percentage of area under rice
and wheat.

83. Answer: (d)


Explanation:
Inverted Yield Curve:
● A yield curve is a graph that depicts the yields on the bonds, ranging from short-term debt,
such as one month, to longer-term debt, such as 30 years.
Types of Yield Curves and their interpretation:
● The yield on the bonds depends upon the risk involved. Higher the risks, higher would be
the yields.
● Normal Yield Curve: Normally, the yield on the short-term maturity bonds is lower than
that of the long-term maturity bonds. This can be attributed to increased risk in the longer
term (say 30 years). A normal yield curve indicates yields on the longer-term bonds may
continue to rise, responding to the periods of economic expansion.
● Inverted Yield Curve: When there are signs of slowdown in an economy, it would mean that
the economy faces risk in the short-term. However, in the long-term, the economy may come
back to normalcy. Hence, due to this, the yield on the short-term bonds becomes higher
than the yields of the long-term bonds. (Inverted Yield Curve). Hence, an inverted yield
curve points towards a probable economic recession.

RAUSIAS-0673 39
https://upscpdf.com/ https://upscpdf.com/
https://t.me/visioniastestseries2024
https://upscmaterial.online/

https://upscpdf.com/ https://upscpdf.com/

84. Answer: (b)


Explanation:
About the Exchange-Traded Funds (ETFs):
• An Exchange-Traded Fund (ETF) is a type of pooled investment security that operates much
like a mutual fund.
• It can be structured to track anything, from the price of an individual commodity, to a
large and diverse collection of securities.
• The ETFs can, thus, contain many types of investments, including stocks, commodities,
bonds, or a mixture of investment types.
• The ETF share prices fluctuate all day, as the ETF is bought and sold. This is different from
the mutual funds, which only trade once a day after the market closes.

85. Answer: (a)


Explanation:
Claudia Goldin’s ‘U’-shaped Curve Hypothesis on ‘Female Labour Participation Rate
(FLPR)’:
Claudia Goldin (Recently, the Nobel Memorial Prize in Economic Sciences has been awarded to
her) had stated that the Female Labour Force Participation Rate (FLFPR) exhibits a U-shape
during the process of economic development. The downward trend in the ‘U’ was due to the
rise in the household incomes, because of expansion of the markets and shift from farm
activities to factory work.
However, when the educational level rises and the economy moves towards the service sector, as
the value of women’s time in the market increases further, they move back into the paid labour
force.

RAUSIAS-0673 40
https://upscpdf.com/ https://upscpdf.com/
https://t.me/visioniastestseries2024
https://upscmaterial.online/

https://upscpdf.com/ https://upscpdf.com/

Gender inequality at work: Goldin’s findings also showed that despite increasing FLPR,
with the rise of the service sector, the gender gap in earnings remained high. In fact, the
difference between men and women workers in the same occupation rose with the birth of the
first child.

86. Answer: (b) Recently, Arunachal Pradesh, Assam,


Explanation: Manipur and Nagaland have also initiated
Oil Palm (Elaeis guineensis), originated oil palm plantation programmes on a large
from West Africa, is comparatively a new scale.
crop in India and has the highest vegetable National Mission on Edible Oils - Oil
oil yielding capability per ha. It produces Palm (NMEO-OP): Considering the growing
two distinct oils, viz., palm oil and palm domestic demand for edible oils, the
kernel oil, which are used for culinary, as staggering deficiency, and the cost to the
well as industrial purposes. exchequer on account of imports, the
Palm oil is derived from the fleshy urgency of scaling-up the oil palm area is
mesocarp of the fruit, which contains of national interest and does not need any
about 45%-55% of oil. Palm kernel oil, over emphasis.
obtained from the kernel of oil palm, is the Therefore, to fulfil the national interest, the
source of the lauric oils. National Mission on Edible Oils - Oil Palm
With quality planting material, irrigation (NMEO-OP) has been approved with the
and proper management, there is a aim to enhance the edible oilseeds
potential of 20 MT -25 MT fresh fruit production and oils availability in the
bunches (FFBs) per ha, after attaining the country by harnessing oil palm area
age of 8 years -9 years. In comparative expansion, increasing the crude palm oil
terms, the yield of palm oil is 5 times the (CPO) production and to reduce import
yield of edible oil obtainable from the burden on edible oils. It will focus on
traditional oilseeds. increasing edible oil production from oil
palm.
Andhra Pradesh, Telangana and Kerala are
the major oil palm growing states and The State Department of Agriculture; the
account for 98% of the total production. State Department of Horticulture; the
Central University; the ICAR (The Indian
Karnataka, Tamil Nadu, Odisha, Gujarat
Council of Agricultural Research)-
and Mizoram have also sizable areas under
Institutions; the CDDs (The Crop
oil palm cultivation.
RAUSIAS-0673 41
https://upscpdf.com/ https://upscpdf.com/
https://t.me/visioniastestseries2024
https://upscmaterial.online/

https://upscpdf.com/ https://upscpdf.com/
Development Directorates); the SAUs (The Bengal and the Andaman & Nicobar
State Agricultural Universities); the KVKs islands.
(The Krishi Vigyan Kendras); the Central The Targets of the National Mission on
Agencies/Cooperatives; the oil palm Edible Oils - Oil Palm (NMEO-OP):
processors/associations; DD Around 9 MT of palm oil is imported every
(Doordarshan) Kisan; AIR (All India Radio); year, to the tune of Rs. 40,000 crores,
DD (Doordarshan); and TV channels will be which is around 56 % of the total imports
the implementing stake holders of the of edible oils. At present, against a total
NMEO-OP. potential area of around 28 lakh hectares,
The strategy to implement the proposed only 3.70 lakh hectares is under oil palm
NMEO-OP will include increasing the cultivation.
production of the seedlings by The target fixed for oil palm area
establishment of seed gardens, nurseries of expansion by 2025-26, under NMEO-OP,
oil palm, to assure domestic availability of is given below:
the seedlings, as per the target fixed under To increase the area of oil palm to 10 lakh
the NMEO-OP. hectares, from 3.5 lakh ha during 2019-20,
Improving the productivity of the FFBs by 2025-26 (additional 6.50 lakh ha), of
(Fresh Fruit Bunches), increasing drip which it is targeted 3.22 lakh hectares for
irrigation coverage under oil palm, the general states and 3.28 lakh ha in the
diversification of area from low-yielding North-Eastern (NE) states, with targeted
cereals crops to oil palm, inter-cropping FFBs production of 66.00 lakh tonnes.
during gestation period of 4 years, would To increase crude palm oil production from
provide economic return to the farmers 0.27 lakh tonnes during 2019-20, to 11.20
when there is no production. lakh tonnes by 2025-26.
The scheme would be implemented in a To increase consumer awareness to
mission mode through active involvement maintain the consumption level of
of all the stakeholders. Fund flow would be 19.00 kg/person/annum till 2025-26.
monitored to ensure that the benefit of the
NMEO-OP also offers special assistance of
Mission reaches the targeted beneficiaries
Rs. 70,000 per hectare to the farmers for
in time to achieve the targeted results.
planting material and management. The
NMEO-OP will be implemented in the Mission is also providing Rs. 2,90,000 for
following states. the purchase of harvesting tools to the
NMEO-GC (General Council), headed by farmers for palm oil cultivation and
the Agriculture Minister, would be assistance of Rs. 25 lakhs for establishing
empowered for the inclusion of additional the ‘custom hiring centres’ (CHCs).
potential states/districts, with consultation Under the NMEO-OP, the cost will be
of the ICAR (The Indian Council of shared as 60:40 between the Central
Agricultural Research)/ IIOPR (The Indian Government and the State Government for
Institute of Oil Palm Research), in addition the general states and 90:10 for the NE
to the identified areas, as per the report of states, and 100% for the UTs and for the
the IIOPR, 2020. central agencies.
NMEO-OP: Potential states and districts of
Andhra Pradesh, Chhattisgarh, Goa,
87. Answer: (c)
Gujarat, Maharashtra, Madhya Pradesh,
Explanation:
Mizoram, Karnataka, Kerala, Odisha, Tamil
Nadu, Telangana, Arunachal Pradesh, • ‘Gross Fixed Capital Formation’ (GFCF)
Assam, Bihar, Manipur, Meghalaya, is defined as the gross addition to the
Nagaland, Tripura, Uttar Pradesh, West fixed assets of an economy in a year.

RAUSIAS-0673 42
https://upscpdf.com/ https://upscpdf.com/
https://t.me/visioniastestseries2024
https://upscmaterial.online/

https://upscpdf.com/ https://upscpdf.com/
• Fixed assets are those which are used including the CPSEs (Central Public Sector
repeatedly or continuously in Enterprises), in identifying surplus non-
production processes for more than one core assets and monetizing them
year. Fixed assets include not only professionally and efficiently to maximise
structures, machinery and equipment, their value.
but also cultivable biological resources,
such as trees, plants and animals that 89. Answer: (b)
are used repeatedly or continuously to
Explanation:
produce other products, such as fruits
A Lakhpati Didi is a Self-Help Group
and dairy products.
member who earns an annual household
They also include the intellectual property income of Rupees One Lakh (Rs. 1,00,000)
products, such as software, data base and
or more. This income is calculated for at
products of artistic originals used in
least four agricultural seasons and/or
production.
business cycles, with an average monthly
• The GFCF is not a measure of total income exceeding Rupees Ten Thousand
investment, because only the value of (Rs. 10,000), so that it is sustainable.
additions to the fixed assets is They serve as an inspiration to the
measured, and all kinds of financial community, not solely for their income, but
assets are excluded, as well as stocks of for their transformation journey through
inventories and other operating costs. adopting sustainable livelihood practices
• It does not account for the (farm or non-farm or service), effectively
consumption (depreciation) of the managing resources, and achieving a
fixed capital. decent standard of life.
The eligible women are being provided
88. Answer: (b) interest-free financial assistance of Rs 1-5
Explanation: lakh along with many other benefits.
• The Union Government has set-up the Under this scheme, women will get training
National Land Monetization in plumbing, making LED bulbs, operating
Corporation (NLMC) as a wholly-owned drones etc. so that they can earn well.
Government of India company with an Ministry of the Scheme - Ministry of
initial authorized share capital of Rs. Rural Development
5,000 crores. It will undertake
monetization of surplus land and 90. Answer: (c)
building assets of the Central Public
Explanation:
Sector Enterprises (CPSEs) and other
Section 15 of the Micro, Small and Medium
Government agencies.
Enterprises Development Act (The MSMED
• Surplus land and building assets of the Act), 2006, mandates payments to the
PSUs (Public Sector Undertakings) micro and small enterprises within the
would be transferred to the NLMC. The time as per the written agreement, which
NLMC would then undertake the cannot be more than 45 days. If there is no
monetization of the surplus land and such written agreement, the Section
building assets. As part of such an mandates that the payment shall be made
exercise, the NLMC would either sell within 15 days.
or lease the surplus land and
Where any buyer fails to make payment of
buildings to the private sector.
the amount to the supplier, the buyer shall
The Corporation will also guide and be liable to pay compound interest with
support other government entities, monthly rests to the supplier on that

RAUSIAS-0673 43
https://upscpdf.com/ https://upscpdf.com/
https://t.me/visioniastestseries2024
https://upscmaterial.online/

https://upscpdf.com/ https://upscpdf.com/
amount from the appointed day or, as the • The Crypto-Asset Reporting Framework
case may be, from the date immediately (commonly referred to as CARF) is a
following the date agreed upon, at three global initiative led by the OECD (The
times of the bank rate notified by the Organisation for Economic Co-
Reserve Bank of India. operation and Development) Global
Thus, the proposed amendment to Section Forum on Transparency and Exchange
43B of the Income Tax Act will allow the of Information for Tax Purposes, which
payment as deduction only on payment is intended to promote the automatic
basis. It can be allowed on accrual basis exchange of information between the
only if the payment is within the time countries to tackle emerging tax
mandated under Section 15 of the MSMED evasion risks related to cryptocurrency
Act. and digital assets.
Section 43B of the Act provides for certain
deductions to be allowed only on actual 92. Answer: (b)
payment. Further, the proviso of this Explanation:
Section allows deduction on accrual basis,
• Avdiivka: Russia has registered a big
if the amount is paid by due date of
victory, capturing the town of Avdiivka
furnishing of the return of income.
in Ukraine.
In order to promote timely payments to the
• Cabo Verde: Became the third country
micro and small enterprises, it is proposed
in Africa to be declared malaria-free by
to include payments made to such
the World Health Organization (WHO).
enterprises within the ambit of Section 43B
• Chancay: The upcoming inauguration
of the Act. Accordingly, it is proposed to
of the $3.5 billion deep water port in
insert a new Clause (h) in Section 43B of
Chancay, Peru, marks a significant
the Act to provide that any sum payable by
milestone as the first port in South
the assessee to a micro or small enterprise
America under Chinese management.
beyond the time limit specified in Section
15 of the Micro, Small and Medium • Nova Kakhovka: UNOSAT (The United
Enterprises Development (MSMED) Act Nations Satellite Centre) analysts
(2006) shall be allowed as deduction only have commenced evaluating the
on actual payment. However, it is also aftermath of the Nova Kakhovka dam
proposed that the proviso to Section 43B of collapse in Ukraine in 2023.
the Act shall not apply to such payments.
This amendment will take effect from 1st 93. Answer: (b)
April, 2024, and will accordingly apply to Explanation:
the assessment year 2024-25 and The International Criminal Court (ICC):
subsequent assessment years. (Changes • It is a permanent international court
introduced by budget 2023-24). established to investigate, prosecute
and try the individuals accused of
91. Answer: (b) committing the most serious crimes
Explanation: of concern to the international
• ‘The Markets in Crypto-Assets (MiCA)’ community as a whole, namely the
Regulation institutes uniform EU (The crime of genocide, crimes against
European Union) market rules for humanity, war crimes and the crime
crypto-assets. The regulation covers of aggression.
crypto-assets that are not currently • It is not a substitute for the national
regulated by the existing financial courts. According to the Rome Statute,
services legislation(s). it is the duty of every State to exercise

RAUSIAS-0673 44
https://upscpdf.com/ https://upscpdf.com/
https://t.me/visioniastestseries2024
https://upscmaterial.online/

https://upscpdf.com/ https://upscpdf.com/
its criminal jurisdiction over those Russia, Tajikistan, Uzbekistan, India
responsible for international crimes. and Pakistan.
• The ICC, which tries the individuals, is • Iran joined the Shanghai Cooperation
also different from the International Organization in July, 2023.
Court of Justice, which is the principal
judicial organ of the United Nations for 96. Answer: (B)
the settlement of disputes between the
Explanation:
States.
The Asia-Pacific Economic Cooperation
• The Court is funded by contributions
(APEC):
from the States Parties and by
• APEC membership includes: Australia;
voluntary contributions from the
Brunei Darussalam; Canada; Chile;
governments, international
People's Republic of China; Hong Kong,
organizations, individuals, corporations
China; Indonesia; Japan; Republic of
and other entities.
Korea; Malaysia; Mexico; New Zealand;
• The ICC can investigate and, where
Papua New Guinea; Peru; the
warranted, prosecute and try the
Philippines; the Russian Federation;
individuals only if the State concerned
Singapore; Chinese Taipei; Thailand;
does not, cannot or is unwilling to do so the United States of America.
genuinely.
• In 1993, the former US President, Bill
• The seat of the Court is in The Hague in
Clinton, established the practice of an
the Netherlands. annual APEC Economic Leaders'
Meeting to provide greater strategic
94. Answer: (c) vision and direction for cooperation in
Explanation: the region.
The Senegal’s Pink Lake: Lake Retba, • There are no binding commitments or
better known as Lac Rose (The Pink Lake), treaty obligations.
is located around 35 km from the city of • The commitments are undertaken on a
Dakar, Senegal. It sits in a depression with voluntary basis and capacity building
a shoreline 6.5 metres below the sea level. projects help the members implement
The lake’s waters are virtually devoid of the APEC initiatives.
life, with the exception of a few microscopic Note: India is not a member and has never
algae and bacteria. It is one of the main hosted any APEC Summit.
tourist destinations in the Dakar region,
primarily because of the pink colour of its
97. Answer: (d)
waters.
Explanation:
The Red Sea crisis, precipitated by the
95. Answer: (a)
attacks by Yemen’s Houthi rebels on the
Explanation:
container vessels, has started taking a toll
• BRICS: Brazil, Russia, India, China on the shipping trade.
and South Africa.
With the commercial ships in the Suez
• Ethiopia, Egypt, Iran, Saudi Arabia and Canal coming under attack from the
the United Arab Emirates (UAE) are the Houthi rebels, the shipping lines have
new additions to the BRICS during the decided to re-route the ships via the Cape
15th BRICS Summit 2023. of Good Hope to reach Europe/US East
• The Shanghai Cooperation Organization Coast, which takes 20-plus days more.
(SCO): The People's Republic of
China, Kazakhstan, Kyrgyzstan,
RAUSIAS-0673 45
https://upscpdf.com/ https://upscpdf.com/
https://t.me/visioniastestseries2024
https://upscmaterial.online/

https://upscpdf.com/ https://upscpdf.com/
The Cape of Good Hope is located at the members and guest countries, enabling
southern tip of the Cape Peninsula, which easy discoverability. It is aimed at
is also home to Cape Town, the legislative addressing the existing knowledge gap
capital of South Africa. around the right practices to design, build
The waters near the Cape, where the and deploy population scale DPI (Digital
Atlantic and the Indian Oceans meet, can Public Infrastructure).
be treacherous for the ships. The The G20 is among the most influential
warm Agulhas Current from the east runs groupings in today’s world order, as it
into the cold Benguela Current from the represents around 85% of the global GDP,
north-west. over 75% of global trade and about two-
Dangerous waves from these currents thirds of the world population.
have caused many shipwrecks. According The G20 leaders adopted the Green
to the folklore, these shipwrecks led to the Development Pact through the New Delhi
legend of the Flying Dutchman, a ghost Leaders’ Declaration, which demonstrated
ship doomed to sail the oceans forever after their collective commitment towards
being lost in a severe storm near the Cape. addressing crucial matters, such as the
protection and conservation of forests and
98. Answer: (d) ecosystems, agreement on global land
restoration targets, and action against
Explanation:
marine plastic litter, along with recognizing
‘The Deccan High-Level Principles’ and
the power of collaboration in environmental
the International Millets and Other Ancient
action.
Grains Research Initiative are the
‘The Global Biofuel Alliance’ is a shining
outcomes of G20.
example of the spirit of constructive
The Deccan High-Level Principles
collaboration that characterized India’s
demonstrated the G20 collective
G20 Presidency.
responsibility in reinforcing and
complementing the efforts across the
geographies in response to global food 99. Answer: (b)
security crises by means of facilitating Explanation:
humanitarian assistance to the countries The Black Sea is the largest inland body of
and populations in vulnerable situations; water, and is also the
enhancing availability and access to largest meromictic basin, which means
nutritious food and strengthening food that the movement of water between the
safety nets; strengthening policies and lower and the upper layers of the sea is
collaborative actions for climate resilient rare.
and sustainable agriculture and food This creates considerable temperature and
systems; strengthening resilience and nutrient differences between these layers,
inclusivity in agriculture and food value with the lower layers being absolutely free
chains; promoting the one health of oxygen and anoxic.
approach; accelerating innovation and the This complex water chemistry is fueled by
use of digital technology and scaling up extensive freshwater inputs from multiple
responsible public and private investments large rivers and rainfall, with salt water
in agriculture. exchanges with the Aegean Sea only
‘The Global Digital Public Infrastructure through the Bosphorus and the
Repository’ (GDPIR) is an initiative of the Dardanelles Strait.
Indian G20 Presidency. The GDPIR is The Black Sea is bordered by 2 European
designed to be a resource for key lessons Union (EU) countries, viz., Bulgaria and
and knowledge available from the G20

RAUSIAS-0673 46
https://upscpdf.com/ https://upscpdf.com/
https://t.me/visioniastestseries2024
https://upscmaterial.online/

https://upscpdf.com/ https://upscpdf.com/
Romania – as well as Georgia, Moldova, 100. Answer: (a)
Russia, Turkey and Ukraine. Explanation:
The Ebro river discharges into The North Atlantic Treaty Organization
the Mediterranean Sea. (NATO) was created in 1949.
Note: Recently, the ‘Black Sea Grain Sweden became the latest country to join
Initiative’ has been launched. It helps the the Alliance in 2024. NATO's "open door
people in need across the globe by directly policy" is based on Article 10 of its
delivering desperately-needed grains to the founding treaty.
lower-income countries and bringing down Any decision to invite a country to join the
the food prices. Alliance is taken by the North Atlantic
Council on the basis of consensus among
all Allies.
Ukraine is not a NATO member. Ukraine is
a NATO partner country, which means that
it cooperates closely with NATO but it is
not covered by the security guarantee in
the Alliance’s founding treaty.

RAUSIAS-0673 47
https://upscpdf.com/ https://upscpdf.com/
https://t.me/visioniastestseries2024
https://upscmaterial.online/

https://upscpdf.com/ https://upscpdf.com/

https://upscpdf.com/ https://upscpdf.com/
https://t.me/visioniastestseries2024

You might also like